Как решать уравнения из комбинаторики

Конспект урока на тему «Решение комбинаторных уравнений» (10 класс)

Сочетаниями без повторений занимался еще великий Паскаль. Он предложил специальную таблицу значений сочетаний без повторений.

Значения представлены в табл. которая называется треугольником Паскаля.

Этот треугольник удивительно красив своей математической красотой, и в его числах можно при желании отыскать различные закономерности. Его можно представить несколько иначе – в виде [26]: равнобедренного треугольника (рис. 10).

Как решать уравнения из комбинаторики

Рис. 10. Треугольник Паскаля

Здесь каждое число, кроме единиц на боковых сторонах, является суммой двух чисел, стоящих над ним. Поэтому:

(приводим к общему знаменателю)

(выносим n ! за скобку в знаменателе)

Из этого соотношения и вытекает эффективный способ рекуррентного вычисления значений биномиальных коэффициентов.

Докажем соотношение 1)

Это может использоваться при вычислениях, например, вместо можно вычислить .

Докажем соотношение 2)

Имеется формула, называемая биномом Ньютона, которая использует выражения числа сочетаний с повторениями

где а, b – действительные или комплексные числа.

Коэффициенты называются биномиальными.

Докажем формулу бинома Ньютона по индукции. Доказательство по индукции предполагает:

1) базис индукции – доказательство того, что формула верна для конкретного n , например, для n =1. В нашем случае мы убедились, что формула верна для n =2,3,4. Убедимся, что она верна и для n =1.

2) индукционный шаг. Предполагая, что формула верна для некоторого n , убеждаются, что тогда она верна и для n +1.

3) при истинности шагов 1 и 2 заключают, что формула верна для любого n .

Приступим к индукционному шагу.

Возьмем выражение и получим из него выражение для n +1. Очевидно, что это можно сделать путем умножения на a + b :

Преобразуем полученное выражение:

Для выполнения индукционного шага необходимо показать, что это выражение равно выражению:

Рассмотрим подвыражение выражения (1): и заменим i на i -1.

Получим , т.е. одинаковые коэффициенты перед выражениями , для числа сочетаний в первом и втором подвыражении выражения (1).Это позволит вынести за скобку. Но тогда в не учтен n -й член подвыражения (суммирование идет до n ): тогда, учитывая его, получаем:

Нетрудно видеть, что можно заменить на , кроме того, мы уже доказали, что , поэтому: , что, очевидно, равно выражению:

По индукции получаем, что формула бинома Ньютона верна для любого n .

С использованием бинома Ньютона докажем следствие №1 о количестве подмножеств множества из n элементов:

Рассмотрим следствие №2: .

На использовании бинома Ньютона основано понятие производящей функции – функции, позволяющей получать комбинаторные числа без вычисления факториала:

. Здесь – функция, производящая биномиальные коэффициенты.

При n =1 получаем 1+ x , т.е. (коэффициент перед 1), (коэффициент перед x ).

При n =2 получаем (1+ x ) 2 =1+2 x + x 2 , т.е. и т.д.

Решение комбинаторных уравнений

В комбинаторике тоже могут решаться уравнения, особенностью которых является то, что неизвестная принадлежит множеству натуральных чисел. Например, уравнения вида , xN , где N – множество натуральных чисел или вида:

При решении комбинаторных уравнений часто необходимо уметь выполнять действия с факториалами типа:

Например, в задаче о сравнении пар записей в базе данных из n записей:

, – что и требовалось доказать.

В комбинаторике рассматриваются и другие типовые комбинаторные комбинации, например, разбиения n -элементного множества на k подмножеств, которые называются блоками разбиения. В информатике вычисления на конечных математических структурах часто называют комбинаторными вычислениями, и они требуют комбинаторного анализа для установления свойств и оценки применимости используемых алгоритмов. На рис. 11 приведен один из возможных вариантов классификации основных комбинаций.

Как решать уравнения из комбинаторики

Рис. 11. Основные комбинации

Комбинаторные задачи могут быть решены, например, системой компьютерной математики Matematica (3,4) фирмы Wolfram Research , Inc . – пакет расширения «Дискретная математика» ( DiscreteMath ) – комбинаторика и ее функции ( Combinatorica , CombinatorialFunctions ): функции перестановок и сочетаний и др.

Пример 1. Решить уравнение

Как решать уравнения из комбинаторики

Как решать уравнения из комбинаторики

и представим правую часть в виде

Как решать уравнения из комбинаторики,

Как решать уравнения из комбинаторики

Как решать уравнения из комбинаторики

Как решать уравнения из комбинаторикиоткуда следует

Как решать уравнения из комбинаторики

Как решать уравнения из комбинаторики

x + 3 = 11 и x = 8.

Пример 2. Решить уравнение

Как решать уравнения из комбинаторики

Решение. По условию x – целое число, удовлетворяющее неравенством Как решать уравнения из комбинаторикиПерепишем уравнение в виде

Как решать уравнения из комбинаторики

Как решать уравнения из комбинаторики

Как решать уравнения из комбинаторики

Как решать уравнения из комбинаторики

Как решать уравнения из комбинаторики

Как решать уравнения из комбинаторики

откуда, после упрощений, получаем

Как решать уравнения из комбинаторики

Как решать уравнения из комбинаторики> 4

Пример 3. Решить систему уравнений

Как решать уравнения из комбинаторики

Решение. Из второго уравнение находим

Как решать уравнения из комбинаторикиРешая последнее уравнение, получаем Как решать уравнения из комбинаторикиНо так как Как решать уравнения из комбинаторикине пригодно к решению уравнения, значит x = 18.

Подставляя x = 18 в первое уравнение системы, найдем

Как решать уравнения из комбинаторики

Как решать уравнения из комбинаторики

18 – y = y + 2, y = 8.

Итак, x = 18, y = 8.

Пример 4. Решить систему уравнений

Как решать уравнения из комбинаторики

Решение. Перепишем систему уравнений в виде

Как решать уравнения из комбинаторикиили, после упрощений получим

Как решать уравнения из комбинаторикиоткуда следует x = 2, y = 6.

Решите уравнение (22–25) .

1)Как решать уравнения из комбинаторики=42;

ОДЗ: хКак решать уравнения из комбинаторикиN; x > 2

Как решать уравнения из комбинаторики= 42

Как решать уравнения из комбинаторики

Как решать уравнения из комбинаторики

Как решать уравнения из комбинаторики=-6( исключить – не входит в ОДЗ); Как решать уравнения из комбинаторики=7

Как решать уравнения из комбинаторики=56х;

ОДЗ: хКак решать уравнения из комбинаторикиN; x > 3

Как решать уравнения из комбинаторики= Как решать уравнения из комбинаторики

Как решать уравнения из комбинаторики

(Как решать уравнения из комбинаторики

Как решать уравнения из комбинаторики((Как решать уравнения из комбинаторики

Как решать уравнения из комбинаторикиили Как решать уравнения из комбинаторики-3Как решать уравнения из комбинаторики

Как решать уравнения из комбинаторики1 =0(исключить) или х 2 =-6 (исключить); х 3 =9 (входит в ОДЗ).

3)Как решать уравнения из комбинаторики=30;

ОДЗ: хКак решать уравнения из комбинаторикиN; x+1 > 2; х > 1

Как решать уравнения из комбинаторики= Как решать уравнения из комбинаторики

Как решать уравнения из комбинаторики

Как решать уравнения из комбинаторики

Как решать уравнения из комбинаторики=-6( исключить – не входит в ОДЗ); Как решать уравнения из комбинаторики=5.

4) 5Как решать уравнения из комбинаторики=Как решать уравнения из комбинаторики;

ОДЗ: Как решать уравнения из комбинаторики Как решать уравнения из комбинаторикихКак решать уравнения из комбинаторики

Как решать уравнения из комбинаторики; Как решать уравнения из комбинаторики=Как решать уравнения из комбинаторики

Как решать уравнения из комбинаторики= Как решать уравнения из комбинаторики

Как решать уравнения из комбинаторики= Как решать уравнения из комбинаторики

(20(х-2)-(х+1)(х+2))Как решать уравнения из комбинаторикихКак решать уравнения из комбинаторики

(20х-40-х 2 +2х+х+2)=0 или х=0 или х-1=0

х 2 +3х-20х+42=0 х 1 =0 х 2 =1

х 2 -17х+42=0 корни 0 и 1 не входят в ОДЗ

Как решать уравнения из комбинаторики= 21 ОДЗ: хКак решать уравнения из комбинаторикиN; x-3 > 2 ; x > 3

Как решать уравнения из комбинаторики= Как решать уравнения из комбинаторики

Как решать уравнения из комбинаторики

Как решать уравнения из комбинаторики

Как решать уравнения из комбинаторики— 7х + 12 – 42 = 0

Как решать уравнения из комбинаторики— 7х – 30 = 0

х 1 =10 х 2 = — 3 (не входит в ОДЗ)

2) Как решать уравнения из комбинаторики; ОДЗ: хКак решать уравнения из комбинаторикиN; x > 3

Как решать уравнения из комбинаторики= Как решать уравнения из комбинаторики

Как решать уравнения из комбинаторики= Как решать уравнения из комбинаторики

4х(х-2)(х-1) = 6Как решать уравнения из комбинаторики

х(4х 2 – 12х+8-30х+90)=0

х=0 или 4х 2 – 42х + 98 = 0

2х 2 – 21х + 49 = 0

Как решать уравнения из комбинаторики= 15(х-1) ОДЗ: хКак решать уравнения из комбинаторикиN; x > 3

Как решать уравнения из комбинаторики= 15(х-1)

Как решать уравнения из комбинаторики= (х-1)х х 1 = 0 или х 2 = 1 — не входят в ОДЗ

Как решать уравнения из комбинаторики= Как решать уравнения из комбинаторикиОДЗ: хКак решать уравнения из комбинаторикиN; x > 4

Как решать уравнения из комбинаторики= Как решать уравнения из комбинаторики

4(х-2)! = 24Как решать уравнения из комбинаторики

х 1 =12; х 2 = — 7(не входит в ОДЗ)

Как решать уравнения из комбинаторики= 43 ОДЗ: хКак решать уравнения из комбинаторикиN; x > 5

Как решать уравнения из комбинаторики= 43

Как решать уравнения из комбинаторики

х 1 =10; х 2 = 3 (не входит в ОДЗ)

Как решать уравнения из комбинаторики= 89 ОДЗ: хКак решать уравнения из комбинаторикиN; x > 7

Как решать уравнения из комбинаторики

х 2 – 11х – 60 = 0

х 1 =15; х 2 = — 4(не входит в ОДЗ)

Как решать уравнения из комбинаторики+ Как решать уравнения из комбинаторики= 162 ОДЗ: хКак решать уравнения из комбинаторикиN; x > 1

Как решать уравнения из комбинаторики= 162

Как решать уравнения из комбинаторики= 162

2Как решать уравнения из комбинаторики

24х + х 2 + 7х + 12 – 324 = 0

х 2 + 31х – 312 = 0

х 1 =8; х 2 = — 39(не входит в ОДЗ)

Как решать уравнения из комбинаторики= Как решать уравнения из комбинаторики

ОДЗ: Как решать уравнения из комбинаторикиx > 4

Как решать уравнения из комбинаторики= Как решать уравнения из комбинаторики

Как решать уравнения из комбинаторики= Как решать уравнения из комбинаторики

(х-2)(х-1)х = 0 или (х-3)-45 = 0

х 1 =2; х 2 = 1 х 3 =0 — не входят в ОДЗ х 4 = 48

Как решать уравнения из комбинаторики= 42 ОДЗ: хКак решать уравнения из комбинаторикиN; x > 4

Как решать уравнения из комбинаторики= 12

Как решать уравнения из комбинаторики= 12 х 2 – х – 12 = 0 х 1 =4; х 2 = — 3(не входит в ОДЗ) Ответ: 4.

Как решать уравнения из комбинаторики= 90 ОДЗ: Как решать уравнения из комбинаторики Как решать уравнения из комбинаторикиКак решать уравнения из комбинаторики

Как решать уравнения из комбинаторики= 90

х 1 =10; х 2 = — 9(не входит в ОДЗ)

Как решать уравнения из комбинаторики= 132 ОДЗ: Как решать уравнения из комбинаторики

Как решать уравнения из комбинаторики= 132

Как решать уравнения из комбинаторики= 132

x 2 +3 x +2–132 = 0

х 1 =10; х 2 = — 13(не входит в ОДЗ)

Как решать уравнения из комбинаторики= 110 ОДЗ: Как решать уравнения из комбинаторики

Как решать уравнения из комбинаторики= 110

Как решать уравнения из комбинаторики= 110

x 2 +3 x +2– 110 = 0

x 2 +3 x – 108 = 0

х 1 =9; х 2 = — 12(не входит в ОДЗ)

Как решать уравнения из комбинаторикиОДЗ: Как решать уравнения из комбинаторики

Как решать уравнения из комбинаторики

Как решать уравнения из комбинаторики

Как решать уравнения из комбинаторики Как решать уравнения из комбинаторикиКак решать уравнения из комбинаторики

Как решать уравнения из комбинаторикирешаем методом сложения — 5у = -30; у = 6

Как решать уравнения из комбинаторикиОДЗ: Как решать уравнения из комбинаторикиКак решать уравнения из комбинаторики; уКак решать уравнения из комбинаторики

Как решать уравнения из комбинаторики Как решать уравнения из комбинаторикиКак решать уравнения из комбинаторики

Как решать уравнения из комбинаторики Как решать уравнения из комбинаторикиКак решать уравнения из комбинаторики

Как решать уравнения из комбинаторики

(х-3)(х-2)(х-1) = 3Как решать уравнения из комбинаторики

4) Как решать уравнения из комбинаторики

Как решать уравнения из комбинаторики

Как решать уравнения из комбинаторики

Как решать уравнения из комбинаторики

Как решать уравнения из комбинаторики

Сколько двузначных чисел можно составить из цифр 1. 3, 5, 8, 9 так, чтобы в каждом числе не было одинаковых цифр?

Из 6 открыток надо выбрать 3. Сколькими способами это можно сделать?

Содержание
  1. Решение комбинаторных уравнений
  2. Комбинаторика — правила, формулы и примеры с решением
  3. Всё о комбинаторике
  4. Комбинаторные задачи с решением
  5. Пример №1
  6. Пример №2
  7. Пример №3
  8. Пример №4
  9. Пример №5
  10. Пример №6
  11. Пример №7
  12. Пример №8
  13. Пример №9
  14. Пример №10
  15. Пример №11
  16. Пример №12
  17. Пример №13
  18. Пример №14
  19. Пример №15
  20. Пример №16
  21. Правила суммы и произведения
  22. Пример №17
  23. Пример №18
  24. Пример №19
  25. Пример №20
  26. Пример №21
  27. Пример №22
  28. Пример №23
  29. Размещения и перестановки
  30. Пример №24
  31. Пример №25
  32. Пример №26
  33. Пример №27
  34. Пример №28
  35. Пример №29
  36. Пример №30
  37. Пример №31
  38. Комбинации и бином ньютона
  39. Пример №32
  40. Пример №33
  41. Пример №34
  42. Пример №35
  43. Пример №36
  44. Пример №37
  45. Пример №38
  46. Пример №39
  47. Элементы комбинаторики
  48. Арифметика случайных событий
  49. Пример №40
  50. Теорема сложения вероятностей несовместных событий
  51. Зависимые и независимые события. Условная и безусловная вероятности
  52. Пример №41
  53. Теорема умножения вероятностей
  54. Что такое комбинаторика
  55. Понятие множества
  56. Равенство множеств
  57. Подмножество
  58. Операции над множествами
  59. Комбинаторика и Бином Ньютона
  60. Схема решения комбинаторных задач
  61. Понятие соединения
  62. Правило суммы
  63. Правило произведения
  64. Упорядоченные множества
  65. Размещения
  66. Пример №42
  67. Пример №43
  68. Пример №44
  69. Пример №45
  70. Перестановки
  71. Пример №46
  72. Пример №47
  73. Пример №48
  74. Сочетания без повторений
  75. Вычисление числа сочетаний без повторений с помощью треугольника Паскаля
  76. Пример №49
  77. Пример №50
  78. Бином Ньютона
  79. Объяснение и обоснование Бинома Ньютона
  80. Свойства биномиальных коэффициентов
  81. Пример №51
  82. Пример №52
  83. Зачем нужна комбинаторика
  84. Правило суммы
  85. Пример №53
  86. Правило произведения
  87. Пример №54
  88. Пример №55
  89. Пример №56
  90. Пример №57
  91. Пример №58
  92. Пример №59
  93. Пример №60
  94. 💡 Видео

Видео:Решите уравнение ➜ ДВИ до ЕГЭСкачать

Решите уравнение ➜ ДВИ до ЕГЭ

Решение комбинаторных уравнений

В комбинаторике тоже могут решаться уравнения, особенностью которых является то, что неизвестная принадлежит множеству натуральных чисел. Например, уравнения вида Как решать уравнения из комбинаторики, xÎN, где N – множество натуральных чисел или вида:

Как решать уравнения из комбинаторики, xÎN (решите!).

При решении комбинаторных уравнений часто необходимо уметь выполнять действия с факториалами типа:

Как решать уравнения из комбинаторики,

Как решать уравнения из комбинаторики.

Например, в задаче о сравнении пар записей в базе данных из n записей:

Как решать уравнения из комбинаторики, – что и требовалось доказать.

В комбинаторике рассматриваются и другие типовые комбинаторные комбинации, например, разбиения n-элементного множества на k подмножеств, которые называются блоками разбиения. В информатике вычисления на конечных математических структурах часто называют комбинаторными вычислениями, и они требуют комбинаторного анализа для установления свойств и оценки применимости используемых алгоритмов. На рис. 11 приведен один из возможных вариантов классификации основных комбинаций.

Как решать уравнения из комбинаторики

Как решать уравнения из комбинаторики

Рис. 11. Основные комбинации

Комбинаторные задачи могут быть решены, например, системой компьютерной математики Matematica (3,4) фирмы Wolfram Research,Inc. –пакет расширения «Дискретная математика» (DiscreteMath) – комбинаторика и ее функции (Combinatorica, CombinatorialFunctions): функции перестановок и сочетаний и др.

ОСНОВНЫЕ ПОНЯТИЯ ТЕОРИИ ГРАФОВ

Способы задания графов

Совокупность множества М с заданным на нем бинарным отношением ТÌМ 2 [9] называется графом

где М – носитель графа – множество вершин, изображаемых точками, Т – сигнатура графа – множество линий, обозначающих отношения и называемых ребрами.

Между элементами М и Т определено отношение инцидентности, т.е. связи между двумя элементами множества М через один элемент множества Т. Примеры графов: отношения отцовства и материнства на множестве людей, отношения подчиненности, карты дорог местности, электрические схемы соединений приборов и т.д.

Как решать уравнения из комбинаторики

Рис. 12. Пример графа «звезда»

Множество линий-ребер в Т задается обозначением пары (i,j), где i,j – инцидентные вершины, отношение Т – «быть связанным».

Теорию графов начали разрабатывать для решения некоторых задач о геометрических конфигурациях, состоящих из точек и линий. В этих задачах несущественно, соединены ли точки конфигурации отрезками прямых или они криволинейны, какова длина линий и другие геометрические характеристики конфигурации. Важно лишь, что каждая линия соединяет какие-либо две из заданных точек [24].

Первые серьезные результаты теории графов связаны с решением задач построения электрических цепей (Г. Кирхгоф) и подсчета числа химических соединений с различными типами молекулярных связей (А. Кэли). Изобразите в виде графа молекулу Как решать уравнения из комбинаторикиКак решать уравнения из комбинаторики

В 30-е годы ХХ века благодаря трудам Д. Кенига [19] теория графов стала развиваться как самостоятельный раздел математики.

Широкое развитие теория графов получила с 50-х годов ХХ века в связи с появлением такой науки, как кибернетика. Графы применяют при анализе функционирования систем. С отдельными компонентами изучаемой системы удобно связывать вершины графа, а с парами взаимодействующих компонент – его ребра. Такой граф называют структурным графом системы.

В некоторых задачах существенно направление ребер графа. Направленные ребра называют дугами, а содержащий их граф – ориентированным (орграфом). Таковым графом может быть изображена диаграмма Хассе. Соответственно граф с неориентированными ребрами называется неориентированным.

Множество ребер может быть пусто. Если же множество вершин пусто, то пусто и множество ребер. Такой граф называется пустым. Линии, изображающие ребра, могут пересекаться на изображении графа, но точки их пересечений не являются вершинами. Различные ребра могут быть инцидентны одной и той же паре вершин, в этом случае они называются кратными. Граф, содержащий кратные ребра, называют мультиграфом (псевдографом). Ребро (дуга) может соединять некоторую вершину саму с собой, такое ребро (дуга) называется петлей. Будем рассматривать конечные графы, содержащие конечные множества вершин и ребер (дуг).

Рассмотрим предложенную фон Нейманом архитектуру ЭВМ, которая состоит из множества устройств М=, где а – устройство ввода, b – арифметическое устройство (процессор), с – устройство управления, d – запоминающее устройство, е – устройство вывода 10.

Информационный обмен между этими устройствами задается графом (рис. 13).

Как решать уравнения из комбинаторики

Рис. 13. Граф, описывающий архитектуру

фон Неймановской ЭВМ

Вершины графа на рис. 13 для удобства изображены кружками, а не точками, как на рис. 11.

Граф можно задать так называемой матрицей смежности, каждой i-ой строке (j-му столбцу) которой однозначно сопоставляют элемент множества М, между которыми выполняется отношение смежности. Две вершины, инцидентные одному ребру, смежны. Два ребра, инцидентные одной вершине, тоже смежны. Тогда каждая клетка bij взаимно однозначно соответствует элементам множества М×М=М 2 . Клетку bij, которая соответствует элементу, принадлежащему бинарному отношению ТÌМ 2 , отмечают, например, единицей, а в остальные клетки записывают нули.

Рассмотрим матрицу смежности В для графа, изображенного на рис. 13. Устройства i,j находятся в отношении Т, если из устройства i информация поступает в устройство j.

Как решать уравнения из комбинаторики

Граф можно задать и с использованием перечисления его дуг, как это сделано на рис. 13:

Граф можно задать в виде так называемого фактор-множества, представленного парами «элемент множества М – подмножество М, представляющее собой окрестность единичного радиуса этого элемента»:

Ориентированный граф может быть задан и матрицей инцидентности А размерностью n×m: A=||aij||, где n=|M|, m=|Т|, у которой

если вершина ai является концом дуги tj;

если вершина ai является началом дуги tj;

если вершина ai не инцидентна дуге tj,

Как решать уравнения из комбинаторики, Как решать уравнения из комбинаторики.

Как решать уравнения из комбинаторики

Так, для ориентированного графа (рис. 14) матрица инцидентности имеет вид:

Как решать уравнения из комбинаторики

Как решать уравнения из комбинаторики

Рис. 14. Некоторый ориентированный граф

В описанном виде матрицы инцидентности применимы только к графам без петель, в случае наличия которых матрицу надо разбить на две полуматрицы: положительную и отрицательную. Ориентированный граф также может быть задан матрицей смежности.

Для графов с кратными ребрами в матрице смежности указывают кратность ребер, например, для графа, изображенного на рис. 15, матрица смежности представляется в виде:

Как решать уравнения из комбинаторики

Такой граф называют мультиграфом.

Как решать уравнения из комбинаторики

Рис. 15. Некоторый ориентированный мультиграф

Граф называется нагруженным, если каждому ребру (дуге) поставлено в соответствие некоторое действительное число (длина дуги, вес дуги, стоимость дуги и т.д.).

Представим в виде графа некоторые бинарные отношения [9]. Отношение Т в множестве М рефлексивно, как мы уже знаем, если для каждого элемента mÎМ справедливо (m,m)ÎТ. На графе это изображается петлей (рис. 16а). На матрице смежности графа с рефлексивным отношением все элементы, лежащие на главной диагонали отмечены единицами.

Отношение во множестве М называется симметричным, если из (mi,mj)ÎТ следует (mi,mj)ÎМ, mi¹mj (рис. 16б). Матрица смежности симметричного отношения симметрична относительно главной диагонали.

В графе, задающем транзитивное отношение Т, для всякой пары дуг таких, что конец первой совпадает с началом второй, существует транзитивно замыкающая дуга, имеющая общее начало с первой и общий конец со второй.

Как решать уравнения из комбинаторики

Рис. 16. Изображения бинарных отношений в виде графа

а) рефлексивное отношение, б) симметричное отношение,

в) транзитивное отношение

Характеристики графов

Подграфом GА графа G= называется граф, в который входит лишь часть вершин графа G, образующих множество А, вместе с ребрами (дугами), их соединяющими.

Так, карта шоссейных дорог Пермской области является подграфом графа «Карта шоссейных дорог Российской Федерации» [18].

Частичным графом GD по отношению к графу G называется граф, содержащий только часть ребер (дуг) графа G.

Так, карта главных дорог России – подграф карты шоссейных дорог России [18].

Если две вершины соединены ребром, то говорят, что каждая вершина инцидентна этому ребру, а соответствующие вершины – смежны (две вершины, инцидентные одному ребру – смежны). Два ребра, инцидентные одной вершине, также смежны.

Маршрут – чередующаяся последовательность вершин и ребер, в которой два соседних элемента инцидентны [26].

Если начальная вершина маршрута равна конечной, то маршрут замкнут, иначе открыт.

Если все ребра различны, то маршрут называется цепью.

Если все вершины (а, значит и ребра) различны, то маршрут называется простой цепью.

Замкнутая цепь – цикл.

Граф без циклов называется ациклическим.

В ориентированном графе цепь называется путем, а цикл – контуром.

Степенью вершины х, обозначаемой deg(х), называют число ребер, инцидентных ей. Если degх=1, то вершина х тупиковая, если degх=0, то вершина изолированная.

Если G – неориентированный граф с n вершинами и m ребрами, а degj – степени j-й вершины, то сумма степеней вершин равна удвоенному количеству ребер:

Как решать уравнения из комбинаторики.

Это следует из того, что каждое ребро добавляет единицу к степени каждой из двух вершин, которое оно соединяет, т.е. добавляет 2 к сумме уже имеющихся вершин. Следствием этого факта является то, что в каждом графе число вершин нечетной степени четно. Для ориентированного графа вводятся понятия полустепень исхода и полустепень захода.

Деревья. Лес.

Граф связен, если любые две его вершины можно соединить цепью. Если граф не связен, то его можно разбить на отдельные связные подграфы, которые называются компонентами связности.

Связный граф, не имеющий циклов (ациклический), называется деревом (рис. 17).

Как решать уравнения из комбинаторики

Рис. 17. Граф-дерево

Деревом может быть задано отношение подчинения в трудовом коллективе, в государстве.

Простейшее дерево состоит из двух вершин, соединенных ребром. Каждый раз, когда добавляется еще одно ребро, в конце его прибавляется также и вершина. Следовательно, дерево с n вершинами имеет n-1 ребро.

В теории графов доказывается, что число различных деревьев, которые можно построить на m вершинах, равно m m -2 . Много деревьев – это лес.

Цикломатическое число.

Пусть G – неориентированный связный граф, имеющий n вершин и m ребер.

Цикломатическим числом связного графа G с n вершинами и m ребрами называется число

Это число имеет интересный физический смысл: оно равно наибольшему числу независимых циклов в графе [18]. При расчете электрических цепей цикломатическое число используется для определения числа независимых контуров.

Рассмотрим примеры подсчета числа независимых циклов.

В графе, состоящем из одной вершины и одного ребра, один цикл (рис. 18а).

В графе, состоящем из одной вершины и трех ребер, три цикла (рис. 18б).

В графе, состоящем из двух вершин и двух ребер, один цикл (рис. 18в).

В графе, состоящем из двух вершин и пяти ребер, четыре цикла (рис. 18г).

В графе, состоящем из трех вершин и трех ребер, один цикл (рис. 18д).

В графе, состоящем из трех вершин и четырех ребер, два цикла (рис. 18е).

В графе, состоящем из четырех вершин и четырех ребер, один цикл (рис. 18ж).

В графе, состоящем из четырех вершин и пяти ребер, два цикла (рис. 18з).

В графе, состоящем из четырех вершин и шести ребер, три цикла (рис. 18и).

Цикломатическое число дерева равно нулю.

Как решать уравнения из комбинаторики

Рис. 18. Примеры циклов в графах

Плоские (планарные) графы.

Граф называется плоским (планарным), если он может быть изображен на плоскости так, что все пересечения ребер являются вершинами (без пересечения рёбер).

Аналогично можно ввести понятие объемного, т.е. трехмерного графа и т.д.

Хроматическое число графа.

Граф G называют р-хроматическим, где р – натуральное число, если его вершины можно раскрасить р различными цветами так, чтобы никакие две смежные вершины не были раскрашены одинаково. Наименьшее число р, при котором граф является р-хроматическим, называют хроматическим числом графа и обозначают l(G). Если l(G)=2, то граф называют бихроматическим. Необходимым и достаточным условием бихроматичности является отсутствие в графе циклов нечетной длины.

Граф на рис. 19а – бихроматический, его вершины «раскрашены» двумя «цветами», обозначенными 0,1.

Как решать уравнения из комбинаторики

Рис. 19. Примеры раскраски графов

Граф на рис. 19б можно «раскрасить» тремя цветами, например, черным (ч), красным (к) и белым (б).

Изоморфизм графов.

Как мы убедились, граф может быть задан различными способами. Он может быть изображен на чертеже, задан матрицей инцидентности, списком ребер или матрицей смежности, фактор-множеством и т.д. Вид чертежа зависит от формы линий и взаимного расположения вершин [24]. Иногда не так легко понять, одинаково ли графы, изображенные разными чертежами. Вид матриц и списка ребер зависит от нумерации вершин и ребер графа. Строго говоря, граф считается полностью заданным, если нумерация его вершин зафиксирована; графы, отличающиеся только нумерацией вершин, называются изоморфными.

Например, графы, изображенные на рис. 20, изоморфны [24].

Как решать уравнения из комбинаторики

Рис. 20. Изоморфные графы а) и б), отличающиеся

только перенумерацией вершин 5®1, 1®5

Если граф ориентированный, то направление дуг в изоморфных графах должно совпадать.

Чтобы узнать, представляют ли две матрицы смежности изоморфные графы, можно произвести всевозможные одинаковые перестановки строк и столбцов.

Если после одной из этих перестановок возникнет матрица, совпадающая с заданной, то сравниваемые графы изоморфны. Для этого требуется максимум n! перестановок, где n – число вершин графа.

Иногда для определения изоморфности определяют параметры обоих графов: число вершин, число ребер, число компонент связности, последовательность степеней вершин в убывающем порядке.

Если какие-то из этих параметров различны, то эти графы различны. Однако если все параметры у двух графов совпали, это не гарантирует изоморфности, то есть это необходимое, но не достаточное условие [24].

Так, на рис. 21 приведены два графа, у которых эти параметры совпадают, и, тем не менее, они различны [24].

Как решать уравнения из комбинаторики

Рис. 21. Пример неизоморфных графов а) и б)

На рис. 22 приведен граф, изоморфный графу «пятиконечная звезда» (см. рис. 12).

Как решать уравнения из комбинаторики

Рис. 22. Граф, изоморфный графу «пятиконечная звезда»

Двудольный граф (биграф, чётный граф) – граф, который может быть представлен двумя непересекающимися подмножествами вершин, причем все ребра соединяют вершины из разных подмножеств.

Псевдограф содержит и ребра, и дуги.

Тривиальный граф содержит только одну вершину.

Иногда граф задают в виде множества образов Г или прообразов Г -1 .

Множеством внутренней устойчивости графа называется подмножество таких его вершин, которые несмежны между собой.

Множеством внешней устойчивости графа называют такое подмножество его вершин, если любая вершина, не принадлежащая этому подмножеству, смежна с вершинами из этого подмножества.

Множество внешней устойчивости, содержащее наименьшее число вершин, называют наименьшим внешне устойчивым множеством, а число элементов этого множества – число внешней устойчивости графа.

Операции над графами.

Полный граф – это граф, в котором все вершины связаны друг с другом. Очевидно, что это аналог универсума в теории множеств. Поэтому, можно ввести операцию дополнения графа до полного, например, в матрице смежности неориентированного графа заменяются нули на единицы и наоборот, исключая главную диагональ.

Вводятся также операции объединения графов, когда объединяются множества вершин и заданных на них отношений; соединение графов, когда находится пересечение указанных множеств.

Используются и такие операции, как удаление вершины, удаление ребра, добавление вершины, добавление ребра.

В настоящее время в ЭВМ графы чаще всего задаются списками смежности и массивом указателей на эти списки [26].

Задачи на графах могут быть решены, например, системой компьютерной математики Matematica (3,4) фирмы Wolfram Research,Inc. – пакет расширения «Дискретная математика» (DiscreteMath) – представление графов, создание графов, свойства графов, алгоритмическая теория графов.

Видео:Комбинаторное уравнениеСкачать

Комбинаторное уравнение

Комбинаторика — правила, формулы и примеры с решением

Комбинаторика — это раздел математики, в котором изучаются способы выбора и размещения элементов некоторого конечного множества на основании определенных условий. Выбранные (или выбранные и размещенные) группы элементов называются соединениями. Если все элементы полученного множества разные, получаем соединения без повторений, а если элементы повторяются — соединения с повторениями.

Содержание:

В комбинаторике перестановка — это упорядоченный набор без повторений чисел.

Перестановкой из n элементов называется любое упорядоченное множество из n данных элементов.

Иными словами, это такое множество, для которого указано, какой элемент находится на первом месте, какой — на втором, . какой — на n-м.

Формула числа перестановок Как решать уравнения из комбинаторики

Как решать уравнения из комбинаторики

Количество различных шестизначных чисел, которые можно составить из цифр 1, 2, 3, 4, 5, 6, не повторяя эти цифры в одном числе, равноКак решать уравнения из комбинаторики

Размещением из n элементов по k называется любое упорядоченное множество из k элементов, состоящее из элементов данного n-элементного множества.

Формулы для нахождения количества соединений с повторениями обязательны только для классов физико-математического профиля.

Формула числа размещений Как решать уравнения из комбинаторики

Как решать уравнения из комбинаторики

Количество различных трехзначных чисел, которые можно составить из цифр 1, 2, 3, 4, 5, 6, если цифры не могут повторяться, равно

Как решать уравнения из комбинаторики

Сочетанием без повторений из n элементов по k называется любое k-элементное подмножество данного n-элементного множества.

Формула числа сочетаний Как решать уравнения из комбинаторики

Как решать уравнения из комбинаторики(по определению считают, чтоКак решать уравнения из комбинаторики

Из 25 учащихся одного класса можно выделить пятерых для дежурства по школе Как решать уравнения из комбинаторикиспособами, то есть Как решать уравнения из комбинаторикиспособами.

Некоторые свойства числа сочетаний без повторений

Как решать уравнения из комбинаторики(в частности, Как решать уравнения из комбинаторики)

Как решать уравнения из комбинаторики

Схема поиска плана решения простейших комбинаторных задач:

Если элемент А можно выбрать т способами, а элемент В — n способами (при этом выбор элемента А исключает одновременный выбор элемента В), то А или В можно выбрать m + n способами.

Если элемент А можно выбрать m способами, а после этого элемент В — n способами, то А и В можно выбрать Как решать уравнения из комбинаторикиспособами.

Как решать уравнения из комбинаторики

Объяснение и обоснование:

Понятие соединения. Правило суммы и произведения:

При решении многих практических задач приходится выбирать из определенной совокупности объектов элементы, имеющие те или иные свойства, размещать их в определенном порядке и т. д. Поскольку в этих задачах речь идет о тех или иных комбинациях объектов, то такие задачи называют комбинаторными. Раздел математики, в котором рассматриваются методы решения комбинаторных задач, называется комбинаторикой. В комбинаторике рассматривается выбор и размещение элементов некоторого конечного множества на основании определенных условий.

Выбранные (или выбранные и размещенные) группы элементов называют соединениями. Если все элементы полученного множества разные, получаем размещения без повторений, а если элементы могут повторяться — размещения с повторениями. В этом параграфе мы рассмотрим соединения без повторений.

Решение многих комбинаторных задач базируется на двух основных правилах — правиле суммы и правиле произведения.

Правило суммы. Если на тарелке лежат 5 груш и 4 яблока, то выбрать один фрукт (грушу или яблоко) можно 9 способами (5 + 4 = 9). В общем виде справедливо такое утверждение:

  • если элемент А можно выбрать m способами, а элемент В — n способами (при этом выбор элемента А исключает одновременный выбор элемента В), то А или В можно выбрать m + n способами.

Уточним содержание этого правила, используя понятие множеств и операций над ними.

Пусть множество А состоит из m элементов, а множество В -из n элементов. Если множества А и В не пересекаются (то есть Как решать уравнения из комбинаторики), то множество А Как решать уравнения из комбинаторикиВ состоит изКак решать уравнения из комбинаторикиэлементов.

Правило произведения. Если в киоске продают ручки 5 видов и тетради 4 видов, то выбрать набор из ручки и тетради (то есть пару — ручка и тетрадь) можно 5æ4 = 20 способами (поскольку с каждой из 5 ручек можно взять любую из 4 тетрадей). В общем виде имеет место такое утверждение:

  • если элемент А можно выбрать m способами, а после этого элемент В — n способами, то А и В можно выбрать Как решать уравнения из комбинаторикиспособами.

Это утверждение означает, что если для каждого из m элементов А можно взять в пару любой из n элементов В, то количество пар равно произведению Как решать уравнения из комбинаторики.

В терминах множеств полученный результат можно сформулировать следующим образом. Если множество А состоит из т элементов, а множество В — из n элементов, то множество всех упорядоченных пар* (а; b), где первый элемент принадлежит множеству А (а ∈ А), а второй  множеству В (b ∈ В), состоит из Как решать уравнения из комбинаторикиэлементов.

Повторяя приведенные рассуждения несколько раз (или, более строго, используя метод математической индукции), получаем, что правила суммы и произведения можно применять при выборе произвольного конечного количества элементов.

Упорядоченные множества:

При решении комбинаторных задач приходится рассматривать не только множества, в которых элементы можно записывать в любом порядке, но и так называемые упорядоченные множества. Для упорядоченных множеств существенным является порядок следования их элементов, то есть то, какой элемент записан на первом месте, какой на втором и т. д. В частности, если одни и те же элементы записать в разном порядке, то мы получим различные упорядоченные множества. Чтобы различить записи упорядоченного и неупорядоченного множеств, элементы упорядоченного множества часто записывают в круглых скобках, например (1; 2; 3) ≠ (1; 3; 2).

Рассматривая упорядоченные множества, следует учитывать, что одно и то же множество можно упорядочить по-разному. Например, множество из трех чисел можно упорядочить по возрастанию: (–5; 1; 3), по убыванию: (3; 1; –5), по возрастанию абсолютной величины числа: (1; 3; –5) и т. д.

* Множество всех упорядоченных пар (а; b), где первый элемент принадлежит множеству А (а ∈ А), а второй — множеству В (b ∈ В), называют декартовым произведением множеств А и В и обозначают А × В. Отметим, что декартово произведение В × А также состоит из m*n элементов.

Заметим следующее: для того чтобы задать конечное упорядоченное множество из n элементов, достаточно указать, какой элемент находится на первом месте, какой на втором, . какой на n-м.

Размещения:

Размещением из n элементов по k называется любое упорядоченное множество из k элементов, состоящее из элементов заданного n-элементного множества.

Например, из множества, содержащего три цифры , можно составить следующие размещения из двух элементов без повторений:

(1; 5), (1; 7), (5; 7), (5; 1), (7; 1), (7; 5).

Количество размещений из n элементов по k обозначается Как решать уравнения из комбинаторики(читается: «А из n по k», A — первая буква французского слова arrangement, что означает «размещение, приведение в порядок»). Как видим,Как решать уравнения из комбинаторики

Выясним, сколько всего можно составить размещений из n элементов по k без повторений. Составление размещения представим себе как последовательное заполнение k мест, которые будем изображать в виде клеточек (рис. 21.1). На первое место можем выбрать один из n элементов данного множества (то есть элемент для первой клеточки можно выбрать n способами).

Если элементы нельзя повторять, то на второе место можно выбрать только один элемент из оставшихся, то есть из n – 1 элементов. Теперь уже два элемента использованы и на третье место можно выбрать только один из n – 2 элементов и т. д. На k-е место можно выбрать только один из n – (k –1) = n – k +1 элементов (см. рис. 21.1).

Как решать уравнения из комбинаторики

Поскольку требуется выбрать элементы и на первое место, и на второе, . и на k-е, то используем правило произведения и получим следующую формулу числа размещений из n элементов по k:

Как решать уравнения из комбинаторики

Например, Как решать уравнения из комбинаторики(что совпадает с соответствующим значением, полученным выше). Аналогично можно обосновать формулу для нахождения числа размещений с повторениями. При решении простейших комбинаторных задач важно правильно выбрать формулу, по которой будут проводиться вычисления. Для этого нужно выяснить следующее:

  1. Учитывается ли порядок следования элементов в соединении?
  2. Все ли заданные элементы входят в полученное соединение?

Если, например, порядок следования элементов учитывается и из n данных элементов в соединении используется только k элементов, то по определению это — размещение из n элементов по k.

После определения вида соединения следует также выяснить, могут ли элементы в соединении повторяться, то есть выяснить, какую формулу необходимо использовать — для количества соединений без повторений или с повторениями.

Примеры решения задач:

Пример:

На соревнования по легкой атлетике приехала команда из 12 спортсменок. Сколькими способами тренер может определить, кто из них побежит в эстафете 4 × 100 м на первом, втором, третьем и четвертом этапах?

Решение:

Количество способов выбрать из 12 спортсменок четырех для участия в эстафете равно количеству размещений из 12 элементов по 4 (без повторений), то есть Как решать уравнения из комбинаторики

Для выбора формулы выясняем ответы на вопросы, приведенные выше. Поскольку для спортсменок важно, в каком порядке они будут бежать, то порядок при выборе элементов учитывается. В полученное соединение входят не все 12 заданных элементов. Следовательно, соответствующее соединение — размещение из 12 элементов по 4 (без повторений, поскольку каждая спортсменка может бежать только на одном этапе эстафеты).

Пример:

Найдите количество трехзначных чисел, которые можно составить из цифр 1, 2, 3, 4, 5, 6, 7, если цифры в числе не повторяются.

Решение:

Количество трехзначных чисел, которые можно составить из семи цифр 1, 2, 3, 4, 5, 6, 7, равно числу размещений из 7 элементов по 3, то естьКак решать уравнения из комбинаторики

Для выбора формулы выясняем, что для чисел, которые мы будем составлять, порядок следования цифр учитывается и не все элементы выбираются (только 3 из заданных семи). Следовательно, соответствующее соединение — размещение из 7 элементов по 3 (без повторений).

Пример:

Найдите количество трехзначных чисел, которые можно составить из цифр 1, 2, 3, 4, 5, 6, 0, если цифры в числе не повторяются.

Выбор формулы проводится таким же образом, как и в задаче 2. Следует учесть, что если число, составленное из трех цифр, начинается цифрой 0, то оно не считается трехзначным. Следовательно, для ответа на вопрос задачи можно сначала из заданных 7 цифр записать все числа, состоящие из 3 цифр (см. задачу 2). Затем из количества полученных чисел вычесть количество чисел, составленных из трех цифр, но начинающихся цифрой 0. В последнем случае мы фактически будем из всех цифр без нуля (их 6) составлять двузначные числа. Тогда их количество равно числу размещений из 6 элементов по 2 (см. решение).

Можно выполнить также непосредственное вычисление, последовательно заполняя три места в трехзначном числе и используя правило произведения. В этом случае для наглядности удобно изображать соответствующие разряды в трехзначном числе в виде клеточек, например так:

Как решать уравнения из комбинаторики

Решение:

Количество трехзначных чисел, которые можно составить из семи цифр (среди которых нет цифры 0), если цифры в числе не повторяются, равно числу размещений из 7 элементов по 3, то есть Как решать уравнения из комбинаторики

Но среди данных цифр есть цифра 0, с которой не может начинаться трехзначное число. Поэтому из размещений из 7 элементов по 3 необходимо исключить те размещения, в которых первым элементом является цифра 0. Их количество равно числу размещений из 6 элементов по 2, то есть Как решать уравнения из комбинаторикиСледовательно, искомое количество трехзначных чисел равно Как решать уравнения из комбинаторики

Пример:

Решите уравнениеКак решать уравнения из комбинаторики

Решение:

ОДЗ: x ∈ N, Как решать уравнения из комбинаторики. Тогда получаем: Как решать уравнения из комбинаторики

На ОДЗ это уравнение равносильно уравнениям:

Тогда x = 0 или x = 5. В ОДЗ входит только x = 5.

Уравнения, в запись которых входят выражения, обозначающие количество соответствующих соединений из x элементов, считаются определенными только при натуральных значениях переменной x. Чтобы выражение Как решать уравнения из комбинаторикиимело смысл, следует выбирать натуральные значения Как решать уравнения из комбинаторики(в этом случае Как решать уравнения из комбинаторикитакже существует и, конечно, Ax 2 ≠ 0). Для преобразования уравнения используем формулы:Как решать уравнения из комбинаторики

Объяснение и обоснование:

Перестановкой из n элементов называется любое упорядоченное множество из n заданных элементов.

Напомним, что упорядоченное множество — это такое множество, для которого указано, какой элемент находится на первом месте, какой на втором, . какой на n-м.

Например, переставляя цифры в числе 236 (в котором множество цифр уже упорядоченное), можно составить такие перестановки без повторений: (2; 3; 6), (2; 6; 3), (3; 2; 6), (3; 6; 2), (6; 2; 3), (6; 3; 2) — всего 6 перестановок* .

Количество перестановок без повторений из n элементов обозначается Как решать уравнения из комбинаторики(P — первая буква французского слова permutation — перестановка). Как видим, Как решать уравнения из комбинаторики= 6.

Фактически перестановки без повторений из n элементов являются размещениями из n элементов по n без повторений, поэтому Как решать уравнения из комбинаторикиПроизведение Как решать уравнения из комбинаторикиобозначается n!. Поэтому полученная формула числа перестановок без повторений из n элементов может быть записана следующим образом:

Как решать уравнения из комбинаторики

*Отметим, что каждая из перестановок определяет трехзначное число, составленное из цифр 2, 3, 6 таким образом, что цифры в числе не повторяются.

Например, Как решать уравнения из комбинаторики(что совпадает с соответствующим значением, полученным выше).

С помощью факториалов формулу для числа размещений без повторений

Как решать уравнения из комбинаторики(1)

запишем в другом виде. Для этого умножим и разделим выражение в формуле (1) на произведение Как решать уравнения из комбинаторикитогда

Как решать уравнения из комбинаторики

Следовательно, формула числа размещений без повторений из n элементов по k может быть записана так:

Как решать уравнения из комбинаторики(2)

Для того чтобы этой формулой можно было пользоваться при всех значениях k, в частности при k = n – 1 и k = n, договорились считать, что

Например, по формуле (2) Как решать уравнения из комбинаторики

Обратим внимание, что в тех случаях, когда значение n! оказывается очень большим, ответы оставляют записанными с помощью факториалов. Например,Как решать уравнения из комбинаторики

Примеры решения задач:

Для выбора формулы при решении простейших комбинаторных задач достаточно выяснить следующее:

  1. Учитывается ли порядок следования элементов в соединении?
  2. Все ли заданные элементы входят в полученное соединение?

Если, например, порядок следования элементов учитывается и все n заданных элементов используются в соединении, то по определению это перестановки из n элементов.

Пример:

Найдите, сколькими способами можно восемь учащихся построить в колонну по одному.

Решение:

Количество способов равно числу перестановок из 8 элементов, то есть Как решать уравнения из комбинаторики

Для выбора соответствующей формулы выясняем ответы на вопросы, приведенные выше. Поскольку порядок следования элементов учитывается и все 8 заданных элементов выбираются, то искомые соединения — это перестановки из 8 элементов без повторений. Их количество можно вычислить по формуле Как решать уравнения из комбинаторики

Пример:

Найдите количество различных четырехзначных чисел, которые можно составить из цифр 0, 3, 7, 9 (цифры в числе не повторяются).

Решение:

Из четырех цифр 0, 3, 7, 9, не повторяя заданные цифры, можно получить Как решать уравнения из комбинаторикиперестановок. Перестановки, начинающиеся с цифры 0, не являются записью четырехзначного числа — их количество Как решать уравнения из комбинаторики. Тогда искомое количество четырехзначных чисел равноКак решать уравнения из комбинаторики

Поскольку порядок следования элементов учитывается и для получения четырехзначного числа надо использовать все элементы, то искомые соединения — это перестановки из 4 элементов. Их количество — Как решать уравнения из комбинаторики. При этом необходимо учесть, что в четырехзначном числе на первом месте не может стоять цифра 0. Таких чисел будет столько, сколько раз мы сможем выполнить перестановки из 3 оставшихся цифр, то есть Как решать уравнения из комбинаторики

Пример:

Имеется десять книг, из которых четыре — учебники. Сколькими способами можно поставить эти книги на полку так, чтобы все учебники стояли рядом?

Решение:

Сначала будем рассматривать учебники как одну книгу. Тогда на полке надо расставить не 10, а 7 книг. Это можно сделать Как решать уравнения из комбинаторикиспособами. В каждом из полученных наборов книг можно выполнить еще Как решать уравнения из комбинаторикиперестановок учебников. По правилу умножения искомое количество способов равноКак решать уравнения из комбинаторики

Задачу можно решать в два этапа. На первом будем условно считать все учебники одной книгой.

Тогда получим 7 книг (6 не учебников + 1 условная книга — учебник). Порядок следования элементов учитывается и используются все элементы (поставить на полку необходимо все книги). Следовательно, соответствующие соединения — это перестановки из 7 элементов. Их количество — Как решать уравнения из комбинаторики.

На втором этапе решения будем переставлять между собой только учебники. Это можно сделать Как решать уравнения из комбинаторикиспособами. Поскольку нам надо переставить и учебники, и другие книги, то используем правило произведения.

Объяснение и обоснование:

1. Сочетания без повторений:

Сочетанием без повторений из n элементов по k называется любое k-элементное подмножество заданного n-элементного множества.

Например, из множества можно составить следующие сочетания без повторений из трех элементов: , , , .

Количество сочетаний без повторений из n элементов по k элементов обозначается символом Как решать уравнения из комбинаторики(читается: «число сочетаний из п по k» или «це из п по k», С — первая буква французского слова combinaison — сочетание). Как видим, Как решать уравнения из комбинаторики

Выясним, сколько всего можно составить сочетаний без повторений из n элементов по k. Для этого используем известные нам формулы числа размещений и перестановок. Составление размещения без повторений из n элементов по k проведем в два этапа. Сначала выберем k разных элементов из заданного n-элементного множества, не учитывая порядок выбора этих элементов (то есть выберем kэлементное подмножество из n-элементного множества — сочетание без повторений из n-элементов по k). По нашему обозначению это можно сделать Как решать уравнения из комбинаторикиспособами. После этого полученное множество из k разных элементов упорядочим. Его можно упорядочить Как решать уравнения из комбинаторикиспособами. Получим размещения без повторений из n элементов по k. Следовательно, количество размещений без повторений из n элементов по k в k! раз больше числа сочетаний без повторений из n элементов по k, то естьКак решать уравнения из комбинаторикиОтсюда Как решать уравнения из комбинаторикиУчитывая, что по формуле (2) Как решать уравнения из комбинаторики, получаем:

Как решать уравнения из комбинаторики(3)

Например, Как решать уравнения из комбинаторикичто совпадает со значением, полученным выше.

Используя формулу (3), можно легко обосновать свойство 1 числа сочетаний без повторений, приведенное в табл. 28.

1) Поскольку Как решать уравнения из комбинаторикито

Как решать уравнения из комбинаторики(4)

Для того чтобы формулу (4) можно было использовать и при k = n, договорились считать, что Как решать уравнения из комбинаторикиТогдаКак решать уравнения из комбинаторики

Заметим, что формулу (4) можно получить без вычислений с помощью достаточно простых комбинаторных рассуждений.

Когда мы выбираем k предметов из n, то n – k предметов мы оставляем. Если же, напротив, выбранные предметы оставим, а другие n – k -выберем, то получим способ выбора n – k предметов из n. Мы получили взаимно-однозначное соответствие способов выбора k и n – k предметов из n. Значит, количество одних и других способов одинаково. Но количество одних — Как решать уравнения из комбинаторики, а других Как решать уравнения из комбинаторики, поэтому Как решать уравнения из комбинаторики.

Если в формуле (3) сократить числитель и знаменатель на (n – k)!, то получим формулу, по которой удобно вычислять Как решать уравнения из комбинаторикипри малых значениях k:

Как решать уравнения из комбинаторики(5)

Например,Как решать уравнения из комбинаторики

2. Вычисление числа сочетаний без повторений с помощью треугольника Паскаля:

Для вычисления числа сочетаний без повторений можно применять формулу (3): Как решать уравнения из комбинаторики, а можно последовательно вычислять соответствующие значения, пользуясь следующим свойством:

Как решать уравнения из комбинаторики(6)

Для обоснования равенства (6) можно записать суммуКак решать уравнения из комбинаторики, используя формулу (3), и после приведения полученных дробей к общему знаменателю получить формулу для правой части равенства (6) (проделайте это самостоятельно). Также формулу (6) можно получить без вычислений с помощью комбинаторных рассуждений.

Как решать уравнения из комбинаторики— это количество способов выбрать k +1 предмет из n + 1. Подсчитаем это количество, зафиксировав один предмет (назовем его «фиксированным»). Если мы не берем фиксированный предмет, то нам нужно выбрать k +1 предмет из n тех, что остались, а если мы его берем, то нужно выбрать из n тех, что остались, еще k предметов. Первое можно сделать Как решать уравнения из комбинаторикиспособами, второеКак решать уравнения из комбинаторикиспособами. Всего как раз Как решать уравнения из комбинаторикиспособов, следовательно,

Как решать уравнения из комбинаторики

Это равенство позволяет последовательно вычислять значения Как решать уравнения из комбинаторикис помощью специальной таблицы, которая называется треугольником Паскаля. Если считать, что Как решать уравнения из комбинаторики, то он будет иметь вид, представленный в табл. 29.

Как решать уравнения из комбинаторики

Каждая строка этой таблицы начинается с единицы и заканчивается единицейКак решать уравнения из комбинаторики

Если какая-либо строка уже заполнена, например третья, то в четвертой строке надо записать на первом месте единицу. На втором месте запишем число, равное сумме двух чисел третьей строки, стоящих над ним левее и правее (поскольку по формуле (6) Как решать уравнения из комбинаторикиНа третьем месте запишем число, равное сумме двух следующих чисел третьей строки, стоящих над ним левее и правее Как решать уравнения из комбинаторики, и т. д. (а на последнем месте снова запишем единицу).

Примеры решения задач:

Обратим внимание, что, как и раньше, для выбора формулы при решении простейших комбинаторных задач достаточно ответить на вопросы:

  1. Учитывается ли порядок следования элементов в соединении?
  2. Все ли заданные элементы входят в полученное соединение?

Чтобы выяснить, является ли заданное соединение сочетанием, достаточно ответить только на первый вопрос (см. схему в табл. 28). Если порядок следования элементов не учитывается, то по определению это сочетание из n элементов по k элементов.

Пример:

Из 12 членов туристической группы надо выбрать трех дежурных. Сколькими способами можно сделать этот выбор?

Решение:

Количество способов выбрать из 12 туристов трех дежурных равно количеству сочетаний из 12 элементов по 3 (без повторений), то естьКак решать уравнения из комбинаторики

Для выбора соответствующей формулы выясняем ответы на вопросы, приведенные выше. Поскольку порядок следования элементов не учитывается (для дежурных неважно, в каком порядке их выберут), то соответствующее соединение является сочетанием из 12 элементов по 3 (без повторений). Для вычисления можно использовать формулы (3) или (5), в данном случае применяем формулу (3):Как решать уравнения из комбинаторики

Пример:

Из вазы с фруктами, в которой лежат 10 разных яблок и 5 разных груш, требуется выбрать 2 яблока и 3 груши. Сколькими способами можно сделать такой выбор?

Решение:

Выбрать 2 яблока из 10 можно Как решать уравнения из комбинаторикиспособами. При каждом выборе яблок груши можно выбрать Как решать уравнения из комбинаторикиспособами. Тогда по правилу произведения выбор требуемых фруктов можно выполнить Как решать уравнения из комбинаторикиспособами. ПолучаемКак решать уравнения из комбинаторики

Сначала отдельно выберем 2 яблока из 10 и 3 груши из 5.

Поскольку при выборе яблок или груш порядок следования элементов не учитывается, то соответствующие соединения — сочетания без повторений.

Учитывая, что требуется выбрать 2 яблока и 3 груши, используем правило произведения и перемножим полученные возможности выбора яблок Как решать уравнения из комбинаторикии груш Как решать уравнения из комбинаторики

Бином Ньютона:

Как решать уравнения из комбинаторики

Поскольку Как решать уравнения из комбинаторики(при x ≠ 0 и a ≠ 0), то формулу бинома Ньютона можно записать еще и так:

Как решать уравнения из комбинаторики

Общий член разложения степени бинома имеет вид

Как решать уравнения из комбинаторики(где Как решать уравнения из комбинаторики). Коэффициенты Как решать уравнения из комбинаторикиназывают биномиальными коэффициентaми.

Свойства биномиальных коэффициентов:

  1. Число биномиальных коэффициентов (а следовательно, и число слагаемых) в разложении n-й степени бинома равно n + 1.
  2. Коэффициенты членов, равноудаленных от начала и конца разложения, равны между собой (поскольку Как решать уравнения из комбинаторики)
  3. Сумма всех биномиальных коэффициентов равна Как решать уравнения из комбинаторикиКак решать уравнения из комбинаторики
  4. Сумма биномиальных коэффициентов, стоящих на четных местах, равна сумме биномиальных коэффициентов, стоящих на нечетных местах.
  5. Для вычисления биномиальных коэффициентов можно воспользоваться треугольником Паскаля, в котором вычисления коэффициентов основываются на формуле Как решать уравнения из комбинаторики

Как решать уравнения из комбинаторики

Объяснение и обоснование:

Бином Ньютона:

Двучлен вида a + x также называют биномом. Из курса алгебры известно, что:

Как решать уравнения из комбинаторики

Можно заметить, что коэффициенты разложения степени бинома Как решать уравнения из комбинаторикипри n = 1, 2, 3 совпадают с числами в соответствующей строке треугольника Паскаля. Оказывается, что это свойство выполняется для любого натурального n, то есть справедлива формула

Как решать уравнения из комбинаторики(7)

Формулу (7) называют биномом Ньютона. Правая часть этого равенства называется разложением степени биномаКак решать уравнения из комбинаторики, а числа Как решать уравнения из комбинаторики(при k = 0, 1, 2, . n) называют биномиальными коэффициентами.

Общий член разложения степени бинома имеет вид

Как решать уравнения из комбинаторики

Обосновать формулу (7) можно, например, с помощью метода математической индукции. (Проведите такое обоснование самостоятельно.)

Приведем также комбинаторные рассуждения для обоснования формулы бинома Ньютона.

По определению степени с натуральным показателем Как решать уравнения из комбинаторики Как решать уравнения из комбинаторики(всего n скобок). Раскрывая скобки, получаем в каждом слагаемом произведение n букв, каждая из которых — а или х. Если, например, в каком-либо слагаемом количество букв x равно k, то количество букв а в нем — n – k, то есть каждое слагаемое имеет вид Как решать уравнения из комбинаторикипри некотором k от 0 до n. Покажем, что для каждого такого k число слагаемых anКак решать уравнения из комбинаторикиравно Как решать уравнения из комбинаторики, откуда после приведения подобных членов и получаем формулу бинома. Произведение Как решать уравнения из комбинаторикиполучаем, взяв букву x из k скобок и букву а из n – k тех скобок, которые остались. Разные такие слагаемые получим путем разного выбора первых k скобок, а k скобок из n можно выбрать именно Как решать уравнения из комбинаторикиспособами. Следовательно, общий член разложения бинома Как решать уравнения из комбинаторикидействительно имеет вид Как решать уравнения из комбинаторикигде k = 0, 1, 2, . n.

Именно из-за бинома Ньютона числа Как решать уравнения из комбинаторикичасто называют биномиальными коэффициентами.

Записывая степень двучлена по формуле бинома Ньютона для небольших значений n, биномиальные коэффициенты можно вычислять с помощью треугольника Паскаля (см. табл. 30).

Например, Как решать уравнения из комбинаторики

Так как Как решать уравнения из комбинаторики, формулу бинома Ньютона можно записать в виде:

Как решать уравнения из комбинаторики(8)

Если в формуле бинома Ньютона (8) заменить x на (–x), то получим формулу возведения в степень разности a – x:

Как решать уравнения из комбинаторики

Например, Как решать уравнения из комбинаторики(знаки членов разложения чередуются!).

Свойства биномиальных коэффициентов:

  1. Число биномиальных коэффициентов (а следовательно, и число слагаемых) в разложении n-й степени бинома равно n + 1, поскольку разложение содержит все степени x от 0 до n (и других слагаемых не содержит).
  2. Коэффициенты членов, равноудаленных от начала и конца разложения, равны между собой, поскольку Как решать уравнения из комбинаторики
  3. Сумма всех биномиальных коэффициентов равнаКак решать уравнения из комбинаторики

Для обоснования полагаем в равенстве (7) значения a = x = 1 и получаем:

Как решать уравнения из комбинаторики

Например, Как решать уравнения из комбинаторики

4. Сумма биномиальных коэффициентов, стоящих на четных местах, равна сумме биномиальных коэффициентов, стоящих на нечетных местах.

Для обоснования возьмем в равенстве (7) значения a = 1, x = –1:

Как решать уравнения из комбинаторики

Тогда Как решать уравнения из комбинаторики

Примеры решения задач:

Пример:

По формуле бинома Ньютона найдите разложение степениКак решать уравнения из комбинаторики.

Для нахождения коэффициентов разложения можно использовать треугольник Паскаля (табл. 30) или вычислять их по общей формуле. По треугольнику Паскаля коэффициенты равны: 1, 6, 15, 20, 15, 6, 1. Учитывая, что при возведении разности в степень знаки членов разложения чередуются, получаем:

Как решать уравнения из комбинаторики

Для упрощения записи ответа можно избавиться от иррациональности в знаменателях полученных выражений (см. решение) или сначала учесть, что ОДЗ данного выражения: x > 0. Тогда Как решать уравнения из комбинаторикито есть данное выражение можно записать так: Как решать уравнения из комбинаторикии возвести в степень последнее выражение.

Решение:

Как решать уравнения из комбинаторики

Пример:

В разложении степени Как решать уравнения из комбинаторикинайдите член, содержащий Как решать уравнения из комбинаторики

Решение:

Как решать уравнения из комбинаторики.

Общий член разложения: Как решать уравнения из комбинаторики

По условию член разложения должен содержать Как решать уравнения из комбинаторики, следовательно, Как решать уравнения из комбинаторикиОтсюда k = 6.

Тогда член разложения, содержащий Как решать уравнения из комбинаторики, равен

Как решать уравнения из комбинаторики

На ОДЗ (b > 0) каждое слагаемое в данном двучлене можно записать как степень с дробным показателем. Это позволит проще записать общий член разложения степени Как решать уравнения из комбинаторики

Как решать уравнения из комбинаторики

(где k = 0, 1, 2, . n), выяснить, какой из членов разложения содержит Как решать уравнения из комбинаторикии записать его. Чтобы упростить запись общего члена разложения, запишем:

Как решать уравнения из комбинаторики

Всё о комбинаторике

Пусть имеется несколько множеств элементов:

Как решать уравнения из комбинаторики

Вопрос: сколькими способами можно составить новое множество Как решать уравнения из комбинаторикивзяв из каждого исходного множества по одному элементу? Ответ на этот вопрос дают следующие рассуждения.

Элемент Как решать уравнения из комбинаторикииз первого множества можно выбрать Как решать уравнения из комбинаторикиспособами, элемент Как решать уравнения из комбинаторикииз второго – s способами, элемент с можно выбрать Как решать уравнения из комбинаторикиспособами и т. д. Пару элементов Как решать уравнения из комбинаторикиможно составить Как решать уравнения из комбинаторикиs способами. Это следует из табл. 1.1, в которой перечислены все способы такого выбора.

Как решать уравнения из комбинаторики

Способы выбора трех элементов аbc перечислены в табл. 1.2.

Как решать уравнения из комбинаторики

В этой таблице Как решать уравнения из комбинаторикистрок и Как решать уравнения из комбинаторикиs столбцов. Поэтому искомое число способов выбора трех элементов аbc равно Как решать уравнения из комбинаторикиs Как решать уравнения из комбинаторики. Продолжая рассуждать подобным образом, получим следующее утверждение.

Основной комбинаторный принцип. Если некоторый первый выбор можно сделать Как решать уравнения из комбинаторики способами, для каждого первого выбора некоторый второй можно сделать s способами, для каждой пары первых двух – третий выбор можно сделать Как решать уравнения из комбинаторики способами и т.д., то число способов для последовательности таких выборов равно Как решать уравнения из комбинаторикиs Как решать уравнения из комбинаторики.

Комбинаторные формулы в прикладных задачах теории вероятностей обычно связывают с выбором Как решать уравнения из комбинаторикиэлементов («выборкой объема Как решать уравнения из комбинаторики») из совокупности, состоящей из Как решать уравнения из комбинаторикиэлементов (элементов «генеральной совокупности»). Различают два способа выбора:

  • а) повторный выбор, при котором выбранный элемент возвращается в генеральную совокупность и может быть выбран вновь;
  • б) бесповторный выбор, при котором выбранный элемент в совокупность не возвращается и выборка не содержит повторяющихся элементов.

При повторном выборе каждый по порядку элемент может быть выбран Как решать уравнения из комбинаторикиспособами. Согласно комбинаторному принципу, такую выборку можно сделать Как решать уравнения из комбинаторикиспособами. Например, повторную выборку объема 2 из трех элементов Как решать уравнения из комбинаторикиможно сделать 3 2 =9 способами: Как решать уравнения из комбинаторикиКак решать уравнения из комбинаторики

В случае бесповторной выборки первый элемент можно выбрать Как решать уравнения из комбинаторикиспособами, для второго остается Как решать уравнения из комбинаторикивозможность выбора, третий элемент можно выбрать Как решать уравнения из комбинаторикиспособами и т.д. Элемент выборки с номером Как решать уравнения из комбинаторикиможно выбрать Как решать уравнения из комбинаторикиспособом. Согласно комбинаторному принципу, общее число бесповторных выборок объема Как решать уравнения из комбинаторикиравно

Как решать уравнения из комбинаторики

Число Как решать уравнения из комбинаторикиназывают числом размещений из Как решать уравнения из комбинаторикиэлементов по Как решать уравнения из комбинаторики.

Например, существует Как решать уравнения из комбинаторикиразмещений из трех элементов Как решать уравнения из комбинаторикипо два: Как решать уравнения из комбинаторикиОтметим, что и в первом случае и во втором выборки отличаются либо составом элементов, либо порядком выбора элементов.

Выделим особо случай, когда один за другим выбраны все Как решать уравнения из комбинаторикиэлементов. В этом случае выборки имеют один и тот же состав (все Как решать уравнения из комбинаторикиэлементов) и отличаются только порядком выбора элементов. Поэтому число

Как решать уравнения из комбинаторики

называют числом перестановок из Как решать уравнения из комбинаторикиэлементов.

Например, пять человек могут встать в очередь Как решать уравнения из комбинаторикиспособами. Три элемента Как решать уравнения из комбинаторикиможно переставить Как решать уравнения из комбинаторикиспособами: Как решать уравнения из комбинаторики

Подсчитаем количество бесповторных выборок объема Как решать уравнения из комбинаторики, которые отличаются друг от друга только составом элементов. Пусть X — число таких выборок. Для каждого набора из Как решать уравнения из комбинаторикиэлементов можно выбрать порядок их расположения Как решать уравнения из комбинаторикиспособами. Тогда Как решать уравнения из комбинаторикиравно числу способов выбрать Как решать уравнения из комбинаторикиразличных элементов и выбрать порядок их расположения, т.е. равно числу размещений из Как решать уравнения из комбинаторикиэлементов по Как решать уравнения из комбинаторики:

Как решать уравнения из комбинаторики

Это число называют числом сочетаний из Как решать уравнения из комбинаторикиэлементов по Как решать уравнения из комбинаторики и обозначают через Как решать уравнения из комбинаторикиЕсли в формуле (1.2) умножить числитель и знаменатель на Как решать уравнения из комбинаторики, то

Как решать уравнения из комбинаторики

Например, сочетаний из четырех элементов Как решать уравнения из комбинаторикипо два существует Как решать уравнения из комбинаторики. Это Как решать уравнения из комбинаторики

Так как из Как решать уравнения из комбинаторики элементов выбрать Как решать уравнения из комбинаторики элементов можно единственным образом, то Как решать уравнения из комбинаторикиоткуда следует, что Как решать уравнения из комбинаторики

Величины Как решать уравнения из комбинаторикиназывают биномиальными коэффициентами. Название связано с формулой бинома Ньютона

Как решать уравнения из комбинаторики

Из формулы (1.3) следует, что

Как решать уравнения из комбинаторики

Биномиальные коэффициенты образуют так называемый треугольник Паскаля, который имеет вид:

Как решать уравнения из комбинаторики

В Как решать уравнения из комбинаторики-й строке треугольника Паскаля располагаются коэффициенты, соответствующие представлению Как решать уравнения из комбинаторикипо формуле (1.3). Треугольником удобно пользоваться для нахождения значений Как решать уравнения из комбинаторики. Это значение находится на пересечении Как решать уравнения из комбинаторики-й строки и Как решать уравнения из комбинаторики-го наклонного ряда. Например, Как решать уравнения из комбинаторики

Биномиальные коэффициенты обладают свойством симметрии:

Как решать уравнения из комбинаторики

Это наглядно демонстрирует треугольник Паскаля. Равенство (1.4) подтверждает тот очевидный факт, что выбор Как решать уравнения из комбинаторики элементов из n равносилен выбору тех Как решать уравнения из комбинаторикиКак решать уравнения из комбинаторики элементов из Как решать уравнения из комбинаторики, которые следует удалить, чтобы остались Как решать уравнения из комбинаторики элементов.

При повторном выборе из Как решать уравнения из комбинаторики элементов число выборок объема Как решать уравнения из комбинаторики, которые отличаются только составом равно Как решать уравнения из комбинаторикиЕще раз подчеркнем, что речь идет о выборках, которые отличаются хотя бы одним элементом, а порядок выбора этих элементов во внимание не принимается. Число таких выборок можно подсчитать следующим образом. Между элементами Как решать уравнения из комбинаторикипоставим разграничительные знаки, например, нули: Как решать уравнения из комбинаторикиТаких знаков (нулей) понадобится Как решать уравнения из комбинаторики. На месте каждого элемента поставим столько единиц, сколько раз предполагается выбрать этот элемент. Например, комбинация Как решать уравнения из комбинаторикиозначает, что элемент Как решать уравнения из комбинаторикивыбран четыре раза, элемент Как решать уравнения из комбинаторикивыбран один раз, элемент Как решать уравнения из комбинаторикине выбран, . элемент Как решать уравнения из комбинаторикивыбран два раза. Заметим, что в такой записи число единиц равно объему выборки Как решать уравнения из комбинаторики. Для перебора всех возможных комбинаций нужно из Как решать уравнения из комбинаторикимест выбрать Как решать уравнения из комбинаторикиместо и поставить на них нули, а на остальных местах разместить единицы. Это можно сделать способами.

Как решать уравнения из комбинаторики

Совокупность из Как решать уравнения из комбинаторики элементов разделить на Как решать уравнения из комбинаторикигрупп по Как решать уравнения из комбинаторикиэлементов соответственно Как решать уравнения из комбинаторикиможно Как решать уравнения из комбинаторикиспособами. Порядок элементов внутри каждой из этих Как решать уравнения из комбинаторикигрупп не имеет значения.

Пусть Как решать уравнения из комбинаторики– множества, число элементов в каждом из которых равно соответственно Как решать уравнения из комбинаторикиСоставить множество B из Как решать уравнения из комбинаторикиэлементов множества А1, Как решать уравнения из комбинаторикиэлементов множества А2, …, Как решать уравнения из комбинаторикиэлементов множества Аk, можно, согласно основному комбинаторному принципу, способами.

Как решать уравнения из комбинаторики

Для безошибочного выбора комбинаторной формулы достаточно последовательно ответить на вопросы в следующей схеме:

Как решать уравнения из комбинаторики

Например, число словарей, необходимых для непосредственного перевода с одного на другой, для пяти языков определяется из следующих рассуждений. Для составления словаря выбираем из пяти языков (Как решать уравнения из комбинаторики= 5) любые два (Как решать уравнения из комбинаторики=2). Выбор бесповторный, причем при выборе важен и состав выбора и порядок выбора. Поэтому искомое число словарей равно Как решать уравнения из комбинаторики

Комбинаторные задачи с решением

Комбинаторика — раздел математики, занимающийся вопросом выбора и расположения элементов некоторого конечного множества в соответствии с заданными условиями.

Рассмотрим примеры задач комбинаторики.

Пример №1

Сколькими способами можно выбрать путь из начала координат 0(0,0) в точку В(6,4), если каждый шаг равен единице, но его можно совершать только вправо или вверх? Сколько таких путей проходит через точку А(2,3)?

Решение. Весь путь занимает 10 шагов (четыре вверх и шесть вправо). Для планирования пути следует решить, какие именно по счету четыре шага следует сделать вверх, а остальные шесть — вправо. Выбор бесповторный и нас интересует только состав выбора. Поэтому в описанных условиях всего путей из точки О в точку В будет Как решать уравнения из комбинаторики

Рассуждая подобным образом легко видеть, что путей из точки О в точку А существует Как решать уравнения из комбинаторикиа путь из точки А в точку В можно выбрать Как решать уравнения из комбинаторикиспособами. По комбинаторному принципу всего путей через точку А существует 10 • 5 = 50.

Пример №2

Сколькими способами можно выбрать путь из начала координат 0(0,0) в точку Как решать уравнения из комбинаторикиесли каждый шаг равен 1, но его можно совершать только вправо или вверх? Сколько таких путей проходит через точку Как решать уравнения из комбинаторики(См. пример 1.1 и исходные данные.)

Исходные данные к задаче 1.1.

Как решать уравнения из комбинаторики

Пример №3

В городе с идеальной прямоугольной планировкой (сеть улиц в этом городе изображена на рис. 1.1) из пункта А выходят Как решать уравнения из комбинаторикичеловек. Половина из них идет по направлению Как решать уравнения из комбинаторикиполовина — по направлению Как решать уравнения из комбинаторикиДойдя до первого перекрестка, каждая группа разделяется так, что половина ее идет по направлению Как решать уравнения из комбинаторикиполовина — по направлению Как решать уравнения из комбинаторикиТакое же разделение происходит на каждом перекрестке. Требуется перечислить перекрестки, на которых окажутся люди после прохождения N улиц (отрезков на рис. 1.1), и сколько людей окажется на каждом из этих перекрестков.

Как решать уравнения из комбинаторики

Решение. Каждый человек пройдет N улиц и окажется на одном из перекрестков Как решать уравнения из комбинаторикиКоординаты перекрестков указаны в предположении, что точка А служит началом координат.

На каждом перекрестке для каждого человека производится выбор из двух возможностей: идти в направлении Как решать уравнения из комбинаторикиили в направлении Как решать уравнения из комбинаторикиПоэтому всего возможных путей будет Как решать уравнения из комбинаторики. Из этого следует, что каждый путь пройдет только один человек.

В пункте Как решать уравнения из комбинаторикиокажется столько человек, сколько различных путей ведет в этот пункт из точки А . Чтобы попасть в пункт Как решать уравнения из комбинаторикинеобходимо из N улиц выбрать бесповторным способом к улиц в направлении Как решать уравнения из комбинаторики. Это можно сделать Как решать уравнения из комбинаторикиспособами.

Ответ. Как решать уравнения из комбинаторики

Пример №4

Сколькими способами можно Как решать уравнения из комбинаторики одинаковых предметов распределить между Как решать уравнения из комбинаторикилицами так, чтобы каждый получил не менее одного предмета?

Решение. Поставим эти предметы в ряд. Между ними будет Как решать уравнения из комбинаторикипромежуток. В любые Как решать уравнения из комбинаторикииз этих промежутков поставим разделяющие перегородки. Тогда все предметы разделятся на Как решать уравнения из комбинаторикинепустых частей. Первую часть передадим первому лицу, вторую — второму и т.д. Выбрать же Как решать уравнения из комбинаторикипромежуток из Как решать уравнения из комбинаторикипромежутка можно Как решать уравнения из комбинаторикиспособами. Заметим, что вообще Как решать уравнения из комбинаторики предметов распределить между Как решать уравнения из комбинаторикилицами можно Как решать уравнения из комбинаторикиспособами.

Ответ. Как решать уравнения из комбинаторики

Пример 1.4.

Сколькими способами можно распределить 6 яблок, 8 груш и 10 слив между тремя детьми? Сколькими способами это можно сделать так, чтобы каждый ребенок получил по меньшей мере одно яблоко, одну сливу и одну грушу?

Решение. Яблоки в соответствии с формулой (1.5) можно распределить Как решать уравнения из комбинаторикиспособами, груши — Как решать уравнения из комбинаторики, а сливы Как решать уравнения из комбинаторикиспособами. По комбинаторному принципу всего способов Как решать уравнения из комбинаторикиЕсли необходимо, чтобы каждый ребенок получил по меньшей мере одно яблоко, одну грушу и одну сливу, то в соответствии с формулой предыдущего примера имеем Как решать уравнения из комбинаторикиспособов.

Пример №5

Сколько цифр в первой тысяче не содержат в своей записи цифры 5?

Решение. Для записи любой из цифр 000, 001, 002, . 999 необходимо трижды выбрать повторным способом одну из десяти цифр, поэтому и получается всего Как решать уравнения из комбинаторикичисел. Если цифру 5 исключить, то выбор можно производить только из девяти цифр: 0, 1,2, 3, 4, 6, 7, 8, 9. Поэтому всего получится Как решать уравнения из комбинаторикичисел в первой тысяче, в записи которых нет цифры 5.

Пример №6

Сколько шестизначных чисел содержат в записи ровно три различных цифры?

Решение. Заметим, что всего шестизначных чисел имеется Как решать уравнения из комбинаторики, так как первая цифра может быть любой (исключая нуль), а остальные пять могут быть выбраны Как решать уравнения из комбинаторикиспособами.

Выбрать три ненулевых цифры можно Как решать уравнения из комбинаторикиспособами. Из выбранных трех цифр можно составить Как решать уравнения из комбинаторикишестизначных чисел, из двух — Как решать уравнения из комбинаторики, а из одной — Как решать уравнения из комбинаторикишестизначное число. По формуле (1.7) получаем, что существует Как решать уравнения из комбинаторикишестизначных чисел, в записи которых есть только три заданные цифры. Поэтому общее число шестизначных чисел, в записи которых имеются три отличные от нуля цифры, равно Как решать уравнения из комбинаторики

Учтем теперь возможность использования нуля. К нулю нужно добавить две цифры, что можно сделать Как решать уравнения из комбинаторикиспособами. Если, например, были выбраны цифры 0, 2, 5, то первой цифрой должна быть 2 или 5. К этой первой цифре в соответствии с формулой (1.7) можно добавить Как решать уравнения из комбинаторикикомбинаций остальных пяти цифр. Тогда всего шестизначных чисел, состоящих из 0, 2, 5 будет Как решать уравнения из комбинаторикиВсего же шестизначных чисел, записанных тремя цифрами, среди которых встречается нуль, ровно Как решать уравнения из комбинаторикиВсего чисел, удовлетворяющих условиям задачи, имеется Как решать уравнения из комбинаторики

Пример №7

В саду есть цветы десяти наименований (розы, флоксы, ромашки и т. д.).

а) Сколькими способами можно составить букет из пяти цветков (не принимая во внимание совместимость растений и художественные соображения)?

б) Сколькими способами можно составить букет из пяти различных цветков?

в) Сколькими способами можно составить букет из пяти цветков так, чтобы в букете непременно было хотя бы по одному цветку двух определенных наименований

Решение. а) Если запрета на повторение цветков нет, то мы имеем дело с повторным выбором и нас интересует только состав. Поэтому по формуле (1.5) получаем Как решать уравнения из комбинаторикиспособа.

б) Если цветы должны быть разными, то способ выбора бесповторный и букет можно составить Как решать уравнения из комбинаторикиспособами.

в) Отберем по одному цветку каждого из двух названных наименований. Три остальных цветка можно выбрать из 10 возможных Как решать уравнения из комбинаторикиспособами.

Ответ. а) 2002; б) 504; в) 220.

Пример №8

Имеется Как решать уравнения из комбинаторикияблок, Как решать уравнения из комбинаторикигруш и Как решать уравнения из комбинаторикиперсиков. Сколькими способами можно их разложить по двум корзинам? Сколькими способами можно это сделать, если в каждой корзине должно быть хотя бы по одному фрукту всех названных видов (полагаем, что фруктов каждого наименования два или больше)?

Решение. Ясно, что яблоки можно разложить Как решать уравнения из комбинаторикиспособом (в первую корзину можно не положить яблок совсем, положить одно яблоко, два яблока, …, все яблоки). Те же рассуждения в отношении груш и персиков дают соответственно Как решать уравнения из комбинаторикикомбинаций. По комбинаторному принципу всего будет Как решать уравнения из комбинаторикиспособов.

При ответе на второй вопрос учтем, что следует по одному яблоку сразу положить в каждую из корзин, а остальные Как решать уравнения из комбинаторикияблока раскладывать произвольным образом (в первую корзину либо не добавляем яблок, либо добавляем одно, либо –– два, …, либо – все Как решать уравнения из комбинаторикияблока). Все это можно сделать Как решать уравнения из комбинаторикиспособами. Те же рассуждения насчет других фруктов и комбинаторный принцип дают следующий результат: Как решать уравнения из комбинаторики

Ответ. Как решать уравнения из комбинаторики

Пример №9

Требуется найти число натуральных делителей натурального числа Как решать уравнения из комбинаторики.

Решение. Разложим Как решать уравнения из комбинаторикина простые множители:

Как решать уравнения из комбинаторики

где Как решать уравнения из комбинаторики– различные простые числа. (Например, Как решать уравнения из комбинаторикиКак решать уравнения из комбинаторики)

Заметим, что при разделении числа Как решать уравнения из комбинаторикина любые два множителя Как решать уравнения из комбинаторикии Как решать уравнения из комбинаторикипростые сомножители распределятся между Как решать уравнения из комбинаторикии Как решать уравнения из комбинаторики. Если сомножитель , Как решать уравнения из комбинаторикив число Как решать уравнения из комбинаторикивходит Как решать уравнения из комбинаторикито разложение (1.8) примет вид:

Как решать уравнения из комбинаторики

Так что разложение Как решать уравнения из комбинаторикина два сомножителя сводится к разделению каждого из чисел Как решать уравнения из комбинаторикина две части, а это можно сделать Как решать уравнения из комбинаторикиспособами.

Ответ. Как решать уравнения из комбинаторики.

Пример №10

Сколькими способами легкоатлет, собираясь на тренировку, может выбрать себе пару спортивной обуви, имея 5 пар кроссовок и 2 нары кед?

Очевидно, что выбрать одну из имеющихся пар обуви, кроссовки или кеды, можно 5 + 2 = 7 способами.

Обобщая, приходим к комбинаторному правилу сложения:

  • если некоторый элемент Как решать уравнения из комбинаторикиможно выбрать Как решать уравнения из комбинаторикиспособами, а элемент Как решать уравнения из комбинаторики(независимо от выбора элемента Как решать уравнения из комбинаторики) — Как решать уравнения из комбинаторикиспособами, то выбрать Как решать уравнения из комбинаторикиилиКак решать уравнения из комбинаторикиможно Как решать уравнения из комбинаторикиспособами.

Это правило справедливо также для трех и более элементов.

Пример №11

В меню школьной столовой предлагается на выбор 4 вида пирожков и 3 вида сока. Сколько разных вариантов выбора завтрака, состоящего из одного пирожка и одного стакана сока, имеется у учащегося этой школы? Как решать уравнения из комбинаторики

Пирожок можно выбрать 4 способами и к каждому пирожку выбрать сок 3 способами (рис. 76). Следовательно, учащийся имеет Как решать уравнения из комбинаторикивариантов выбора завтрака.

Обобщая, приходим к комбинаторному правилу умножения:

  • если некоторый элемент Как решать уравнения из комбинаторикиможно выбрать Как решать уравнения из комбинаторики, способами и после каждого такого выбора (независимо от выбора элемента Как решать уравнения из комбинаторики) другой элемент Как решать уравнения из комбинаторикиможно выбрать Как решать уравнения из комбинаторикиспособами, то пару объектов Как решать уравнения из комбинаторикииКак решать уравнения из комбинаторикиможно выбрать Как решать уравнения из комбинаторикиспособами.

Это правило справедливо также для трех и более элементов.

Пример №12

Сколько трехзначных чисел можно составить из цифр 1, 2, 3, 4, если в числе: 1) цифры не повторяются; 2) цифры могут повторяться?

Как решать уравнения из комбинаторики

Решение:

1) Первую цифру можем выбрать 4 способами (рис.77). Так как после выбора первой цифры их останется три (ведь цифры в нашем случае повторяться не могут), то вторую цифру можем выбрать 3 способами.И наконец, третью цифру можем выбрать из оставшихся двух — то есть 2 способами. Следовательно, количество искомых трехзначных у чисел будет равно Как решать уравнения из комбинаторики.

2) Применим комбинаторное правило умножения. Так как цифры в числе могут повторяться, то каждую из цифр искомого числа можно выбрать 4 способами (рис. 78), и тогда таких чисел будет Как решать уравнения из комбинаторики.

Ответ. 1) 24 числа; 2) 64 числа.

Отметим, что решить подобные задачи без применения комбинаторного правила умножения можно только путем перебора всех возможных вариантов чисел, удовлетворяющих условию задачи. Но такой способ решения является слишком долгим и громоздким.

Пример №13

Сколько четных пятизначных чисел можно составить из цифр 5, 6, 7, 8, 9, если цифры в числе не повторяются?

Решение:

Четное пятизначное число можно получить, если последней его цифрой будет 6 или 8. Чисел, у которых последней является цифра 6, будет Как решать уравнения из комбинаторики(рис. 79),

Как решать уравнения из комбинаторики

а тех, у которых последней является цифра 8, — также 24. По комбинаторному правилу сложения всего четных чисел будет Как решать уравнения из комбинаторики.

Пример №14

Азбука племени АБАБ содержит всего две буквы — «а» и «б». Сколько слов в языке этого племени состоит: 1) из двух букв; 2) из трех букв?

Решение:

1) аа, ба, аб, бб (всего четыре слова); 2) ааа, ааб, аба, абб, ббб, бба, баб, баа (всего восемь слов).

Заметим, что найденное количество слов соответствует комбинаторному правилу умножения. Так как на каждое место есть два «претендента» — «а» и «б», то слов, состоящих из двух букв, будет Как решать уравнения из комбинаторики, а из трех букв — Как решать уравнения из комбинаторики.

Пример №15

В футбольной команде из 11 игроков надо выбрать капитана и его заместителя. Сколькими способами это можно сделать?

Решение:

Капитаном можно выбрать любого из 11 игроков, а его заместителем — любого из 10 оставшихся игроков. Таким образом (по правилу умножения), имеем Как решать уравнения из комбинаторикиразных способов.

Пример №16

В Стране Чудес 10 городов и каждые два из них соединяет авиалиния. Сколько авиалиний в этой стране?

Решение. Так как каждая авиалиния соединяет два города, то одним из них может быть любой из 10 городов, а другим — любой из 9 оставшихся. Следовательно, количество авиалиний равно Как решать уравнения из комбинаторики. Но при этом каждую из авиалиний мы учли дважды. Поэтому всего их будет Как решать уравнения из комбинаторики.

Комбинаторные задачи неразрывно связаны с задачами теории вероятностей, еще одного раздела математики.

В ХIII-ХII в. до н. э. встречаются упоминания о вопросах, близких к комбинаторным. Некоторые комбинаторные задачи решали и в Древней Греции. В частности, Аристоксен из Тарента (IV в. до н. э.), ученик Аристотеля, перечислил различные комбинации длинных и коротких слогов в стихотворных размерах. А Папп Александрийский в IV в. н. э. рассматривал число пар и троек, которые можно получить из трех элементов, допуская их повторения. Некоторые элементы комбинаторики были известны и в Индии во II в. до н. э. Индийцы умели вычислять числа, известные нам как коэффициенты формулы бинома Ньютона. Позднее, в VIII в. н. э., арабы нашли и саму эту формулу, и ее коэффициенты, которые сейчас вычисляют с помощью комбинаторных формул или «треугольника Паскаля».

Свой нынешний вид упомянутые комбинаторные формулы приобрели благодаря средневековому ученому Леви бен Гершону (XIV в.) и французскому математику П. Эригону (XVII в.).

В III в. н. э. сирийский философ Порфирий для классификации понятий составил специальную схему, получившую название «древо Порфирия». Сейчас подобные деревья используются для решения определенных задач комбинаторики в разнообразных областях знаний. Некоторые ранее неизвестные комбинаторные задачи рассмотрел Леонардо Пизанский (Фибоначчи) в своей знаменитой «Книге абака» (1202 г.), в частности, о нахождении наименьшего набора различных гирь, позволяющего взвесить груз с любой целочисленной массой, не превышающей заданного числа. Со времен греческих математиков были известны две последовательности, каждый член которых получали по определенному правилу из предыдущих, — арифметическая и геометрическая прогрессии. А Фибоначчи впервые в одной из задач выразил член последовательности через два предыдущих, используя формулу, которую назвали рекуррентной. В дальнейшем метод рекуррентных формул стал одним из мощнейших для решения комбинаторных задач.

Как ни странно, развитию комбинаторики в значительной степени способствовали азартные игры, которые были очень популярны в XVI в. В частности, вопросами определения разнообразных комбинаций в игре в кости в то время занимались такие известные итальянские математики, как Д. Кардано, H. Тарталья и др. А наиболее полно изучил этот вопрос в XVII в. Галилео Галилей.

Современные комбинаторные задачи высокого уровня сложности связаны с объектами в других отраслях математики: определителями, конечными геометриями, группами, математической логикой и т. п.

Правила суммы и произведения

Вспомните, что в математике любые совокупности называют множествами. Объекты, входящие в множества, называют его элементами. Множества обозначают большими латинскими буквами, а их элементы записывают в фигурных скобках. Считают, что все элементы множества различны.

Например, Как решать уравнения из комбинаторики

Множества бывают конечными и бесконечными. Если множество не содержит ни одного элемента, его называют пустым и обозначают символом Как решать уравнения из комбинаторики

Два множества называют равными, если они состоят из одних и тех же элементов.

Если Как решать уравнения из комбинаторики— часть множества Как решать уравнения из комбинаторикито его называют подмножеством множества Как решать уравнения из комбинаторикии записывают Как решать уравнения из комбинаторикиНаглядно это изображают с помощью диаграммы Эйлера (рис. 135, а). В частности, для числовых множеств правильные такие соотношения:

Как решать уравнения из комбинаторики

Случается, что множества Как решать уравнения из комбинаторикиимеют общие элементы. Если множество Как решать уравнения из комбинаторикисодержит все общие элементы множеств Как решать уравнения из комбинаторикии только их, то множество Как решать уравнения из комбинаторикиназывают пересечением множеств Как решать уравнения из комбинаторикиЗаписывают это так: Как решать уравнения из комбинаторикиДиаграммой Эйлера пересечение изображают, как показано на рисунке 135, б. Множество, содержащее каждый элемент каждого из множеств Как решать уравнения из комбинаторикии только эти

Как решать уравнения из комбинаторики

элементы, называется объединением множеств Как решать уравнения из комбинаторикиЕсли Как решать уравнения из комбинаторики— объединение множеств Как решать уравнения из комбинаторикито пишут Как решать уравнения из комбинаторики(рис. 135, в).

Разницей множеств Как решать уравнения из комбинаторикиназывают множество, состоящее из всех элементов множества Как решать уравнения из комбинаторикине принадлежащих множеству Как решать уравнения из комбинаторикиЕго обозначают Как решать уравнения из комбинаторикиНапример, если Как решать уравнения из комбинаторикиКак решать уравнения из комбинаторики

Говоря «множество», «подмножество», порядок их элементов не учитывают. Говорят, что они не упорядочены. Рассматривают и упорядоченные множества. Так называют множества с фиксированным порядком элементов. Их обозначают не фигурными, а круглыми скобками. Например, из элементов множества Как решать уравнения из комбинаторикиможно образовать 6 трёхэлементных упорядоченных множеств: Как решать уравнения из комбинаторики

Как множества, все они равны, как упорядоченные множества — разные.

Существуют задачи, в которых надо определить, сколько различных подмножеств или упорядоченных подмножеств можно образовать из элементов данного множества. Их называют комбинаторными задачами, а раздел математики, в котором рассматривается решение комбинаторных задач, называют комбинаторикой.

Комбинаторика — раздел математики, посвящённый решению задач выбора и расположения элементов некоторого конечного множества в соответствии с заданными правилами.

Рассмотрим два основных правила, с помощью которых решается много комбинаторных задач.

Пример №17

В городе Как решать уравнения из комбинаторикиесть два университета — политехнический и экономический. Абитуриенту нравятся три факультета в политехническом университете и два — в экономическом. Сколько возможностей имеет студент для поступления в университет?

Решение:

Обозначим буквой Как решать уравнения из комбинаторикимножество факультетов, которые выбрал абитуриент в политехническом университете, а буквой Как решать уравнения из комбинаторики— в экономическом: Как решать уравнения из комбинаторикиПоскольку эти множества не имеют общих элементов, то в делом абитуриент имеет Как решать уравнения из комбинаторикивозможностей для поступления в университет.

Описанную ситуацию можно обобщить в виде утверждения, которое называется правилом суммы.

Если элемент некоторого множества Как решать уравнения из комбинаторикиможно выбрать Как решать уравнения из комбинаторикиспособами, а элемент множества Как решать уравнения из комбинаторикиспособами, то элемент из множества Как решать уравнения из комбинаторикиили из множества Как решать уравнения из комбинаторикиможно выбрать Как решать уравнения из комбинаторикиспособами.

Правило суммы распространяется и на большее количество множеств.

Пример №18

Планируя летний отдых, семья определилась с местами его проведения: в Одессе — 1, в Евпатории — 3, в Ялте — 2, в Феодосии — 2. Сколько возможностей выбора летнего отдыха имеет семья?

Решение:

Поскольку все базы отдыха разные, то для решения задачи достаточно найти сумму элементов всех множеств, о которых говорится: Как решать уравнения из комбинаторикиСледовательно, семья может выбирать отдых из 8 возможных.

Пример №19

От пункта Как решать уравнения из комбинаторикидо пункта Как решать уравнения из комбинаторикиведут три тропинки, а от Как решать уравнения из комбинаторики— две. Сколько маршрутов можно проложить от пункта Как решать уравнения из комбинаторикидо пункта Как решать уравнения из комбинаторики

Решение:

Чтобы пройти от пункта Как решать уравнения из комбинаторикидо пункта Как решать уравнения из комбинаторикинадо выбрать одну из трёх тропинок: 1, 2 или 3 (рис. 136). После этого следует выбрать одну из двух других троп: 4 или 5. Всего от пункта Как решать уравнения из комбинаторикидо пункта Как решать уравнения из комбинаторикиведут 6 маршрутов, потому что Как решать уравнения из комбинаторикиВсе эти маршруты можно обозначить с помощью пар:Как решать уравнения из комбинаторики

Обобщим описанную ситуацию.

Если первый компонент пары можно выбрать Как решать уравнения из комбинаторикиспособами, а . второй — Как решать уравнения из комбинаторикиспособами, то такую пару можно выбрать Как решать уравнения из комбинаторикиспособами.

Это — правило произведения, его часто называют основным правилом комбинаторики. Обратите внимание: речь идёт об упорядоченных парах, составленных из различных компонентов.

Правило произведения распространяется и на упорядоченные тройки, четвёрки и любые другие упорядоченные конечные множества. В частности, если первый компонент упорядоченной тройки можно выбрать Как решать уравнения из комбинаторикиспособами, второй — Как решать уравнения из комбинаторикиспособами, третий — Как решать уравнения из комбинаторикиспособами, то такую упорядоченную тройку можно выбрать Как решать уравнения из комбинаторикиспособами. Например, если столовая на обед приготовила 2 первых блюда — борщ (б) и суп (с ), 3 вторых — котлеты (к), вареники (в), голубцы (г) и 2 десертных — пирожные (п) и мороженое (м), то всего из трёх блюд столовая может предложить 12 различных наборов, поскольку Как решать уравнения из комбинаторики

Как решать уравнения из комбинаторики

Как решать уравнения из комбинаторики

Описанной ситуации соответствует диаграмма, изображённая на рисунке 137. Такие диаграммы называют деревьями.

Пример №20

Сколько разных поездов можно составить из 6 вагонов, если каждый из вагонов можно поставить на любом месте?

Решение:

Первым можно поставить любой из б вагонов. Имеем 6 выборов. Второй вагон можно выбрать из оставшихся 5 вагонов. Поэтому, согласно правилу умножения, два первых вагона можно выбрать Как решать уравнения из комбинаторикиспособами. Третий вагон можно выбрать из 4 вагонов, которые остались. Поэтому три первых вагона можно выбрать Как решать уравнения из комбинаторикиспособами. Продолжая подобные рассуждения, приходим к ответу: всего можно составить Как решать уравнения из комбинаторикиразличных поездов.

Обратите внимание на решение последней задачи. Оно свелось к вычислению произведения всех натуральных чисел от 1 до 6. В комбинаторике подобные произведения вычисляют часто.

Произведение всех натуральных чисел от 1 до Как решать уравнения из комбинаторикиназывают Как решать уравнения из комбинаторикифакториалом и обозначают Как решать уравнения из комбинаторики

Как решать уравнения из комбинаторики

Условились считать, что Как решать уравнения из комбинаторики

Языком теории множеств правила суммы и произведения можно сформулировать следующим образом.

Если пересечение множеств Как решать уравнения из комбинаторикипустое, то количество элементов в их объединении Как решать уравнения из комбинаторикиравно сумме количества элементов множеств Как решать уравнения из комбинаторики

Как решать уравнения из комбинаторики

Если множества Как решать уравнения из комбинаторикиимеют общие элементы, то

Как решать уравнения из комбинаторики

Если множества Как решать уравнения из комбинаторикиконечны, то количество возможных пар Как решать уравнения из комбинаторикиравно произведению количества элементов множеств Как решать уравнения из комбинаторики

Как решать уравнения из комбинаторики

Пример №21

В розыгрыше на первенство города по баскетболу принимают участие команды из 12 школ. Сколькими способами могут быть распределены первое и второе места?

Решение:

Первое место может получить одна из 12 команд. После того, как определён обладатель первого места, второе место может получить одна из 11 команд. Следовательно, общее количество способов, которыми можно распределить первое и второе места, равно Как решать уравнения из комбинаторики

Пример №22

Сколько четырёхзначных чисел можно составить из цифр 0,1, 2, 3, 4, 5, если ни одна цифра не повторяется?

Решение:

Первой цифрой числа может быть одна из 5 цифр 1, 2, 3, 4, 5. Если первая цифра выбрана, то вторая может быть выбрана 5-ю способами, третья — 4-мя, четвёртая — 3-мя. Согласно правилу умножения общее число способов равно:

Как решать уравнения из комбинаторики

Пример №23

Упростите выражение Как решать уравнения из комбинаторики

Решение:

Как решать уравнения из комбинаторикиКак решать уравнения из комбинаторики

Размещения и перестановки

Задача:

Сколькими способами собрание из 20 человек может избрать председателя и секретаря?

Решение:

Председателя можно выбрать 20-ю способами, секретаря — из остальных 19 человек — 19-ю способами. По правилу произведения председателя и секретаря собрания могут выбрать Как решать уравнения из комбинаторикиспособами.

Обобщим задачу. Сколько упорядоченных Как решать уравнения из комбинаторикиэлементных подмножеств можно составить из Как решать уравнения из комбинаторикиразличных элементов? На первое место можно поставить любой из данных Как решать уравнения из комбинаторикиэлементов. На второе место — любой из остальных Как решать уравнения из комбинаторикиэлементов и т. д. На последнее Как решать уравнения из комбинаторикиместо можно поставить любой из остальных Как решать уравнения из комбинаторикиэлементов. Из правила произведения следует, что из данных Как решать уравнения из комбинаторикиэлементов можно получить Как решать уравнения из комбинаторикиКак решать уравнения из комбинаторики-элементных упорядоченных подмножеств.

Например, из 4 элементов Как решать уравнения из комбинаторикиупорядоченных двухэлементных подмножеств можно образовать всего Как решать уравнения из комбинаторикиКак решать уравнения из комбинаторики

Упорядоченое Как решать уравнения из комбинаторики-элементное подмножество Как решать уравнения из комбинаторикиэлементного множества называют размещением из Как решать уравнения из комбинаторикиэлементов Как решать уравнения из комбинаторики Их число обозначают Как решать уравнения из комбинаторики

Из предыдущих рассуждений следует, что Как решать уравнения из комбинаторикии что для любых натуральных Как решать уравнения из комбинаторики

Как решать уравнения из комбинаторики

В правой части этого равенства Как решать уравнения из комбинаторикимножителей. Поэтому результат можно сформулировать в виде такого утверждения.

Число размещений из Как решать уравнения из комбинаторикиэлементов по Как решать уравнения из комбинаторикиравно произведению Как решать уравнения из комбинаторикипоследовательных натуральных чисел, наибольшее из которых Как решать уравнения из комбинаторики

Примеры:

Как решать уравнения из комбинаторики

Пример №24

Сколькими способами можно составить дневное расписание из пяти разных уроков, если класс изучает 10 различных предметов?

Решение:

Речь идёт об упорядоченных 5-элементных подмножествах некоторого множества, состоящего из 10 элементов.

Это размещения. Как решать уравнения из комбинаторики

Ответ. 30 240 способами.

Число размещений из Как решать уравнения из комбинаторикиэлементов по Как решать уравнения из комбинаторикиможно вычислять и по другой формуле: Как решать уравнения из комбинаторики(проверьте самостоятельно).

Размещение Как решать уравнения из комбинаторикиэлементов по Как решать уравнения из комбинаторикиназывают перестановками из Как решать уравнения из комбинаторикиэлементов. Их число обозначают Как решать уравнения из комбинаторики

Например, из трёх элементов Как решать уравнения из комбинаторикиможно образовать 6 различных перестановок: Как решать уравнения из комбинаторикиСледовательно, Как решать уравнения из комбинаторики

Подставив в формулу числа размещений Как решать уравнения из комбинаторикиполучим, что Как решать уравнения из комбинаторики

Число перестановок из Как решать уравнения из комбинаторикиэлементов равно Как решать уравнения из комбинаторики!

Примеры:

Как решать уравнения из комбинаторики

Пример №25

Сколькими способами можно составить список из 10 фамилий?

Решение:

Как решать уравнения из комбинаторики

Ответ. 3 628 800 способами.

Некоторые комбинаторные задачи сводятся к решению уравнений, в которых переменная указывает на количество элементов в некотором множестве или подмножестве. Рассмотрим несколько таких уравнений.

Пример №26

Решите уравнение Как решать уравнения из комбинаторики

Решение:

Пользуясь формулой размещений, данное уравнение можно заменить таким:

Как решать уравнения из комбинаторики

По условию задачи Как решать уравнения из комбинаторики— натуральное число, поэтому Как решать уравнения из комбинаторики— посторонний корень. Следовательно, Как решать уравнения из комбинаторики

Пример №27

Решите уравнение Как решать уравнения из комбинаторики

Решение:

Запишем выражения Как решать уравнения из комбинаторикичерез произведения.

Имеем: Как решать уравнения из комбинаторики

Поскольку по смыслу задачи Как решать уравнения из комбинаторикиПоэтому последнее уравнение можно сократить на произведение Как решать уравнения из комбинаторикиТогда Как решать уравнения из комбинаторики Как решать уравнения из комбинаторикиНо уравнение Как решать уравнения из комбинаторикиудовлетворяет только одно значение: Как решать уравнения из комбинаторики

Пример №28

Команда из трёх человек выступает в соревнованиях по художественной гимнастике, в которых принимают участие ещё 27 спортсменок. Сколькими способами могут распределиться места между членами команды, при условии, что на этих соревнованиях ни одно место не делится?

Решение:

Речь идёт об упорядоченных 3-элементных подмножествах множества, состоящего из 30 элементов. Это — размещения. Как решать уравнения из комбинаторики

Пример №29

Сколькими способами можно разместить на полке 5 дисков?

Решение:

Речь идёт об упорядоченных 5-элементных множествах. Искомое количество способов равно Как решать уравнения из комбинаторики

Ответ. 120 способами.

Пример №30

Изображённое на рисунке 140 кольцо раскрашено в 7 цветов. Сколько существует таких колец, раскрашенных теми же цветами только в других последовательностях?

Решение:

Зафиксируем одну какую-нибудь часть кольца, окрашенную одним цветом, б других частей можно раскрасить Как решать уравнения из комбинаторикиспособами.

Как решать уравнения из комбинаторики

Ответ. 720 колец.

Пример №31

Сколько можно составить различных неправильных дробей, числителями и знаменателями которых есть числа 3,5, 7,9,11,13?

Решение:

Способ 1. Дробей, у которых числитель не равен знаменателю, можно составить Как решать уравнения из комбинаторикито есть Как решать уравнения из комбинаторикиИз этих дробей только половина — неправильных, то есть — 15.

Неправильными являются также дроби, у которых числитель равен знаменателю. Таких дробей в нашем случае 6. Итак, всего можно составить Как решать уравнения из комбинаторики(дробь).

Способ 2. Если знаменатель неправильной дроби 3, то его числителями могут быть все 6 данных чисел. Если знаменатель 5, то числителями неправильной дроби могут быть 5 чисел (5, 7, 9, 11, 13) и т.д. Наконец, если знаменатель — число 13, то существует только 1 неправильная дробь, со знаменателем 13. Всего таких неправильных дробей существует Как решать уравнения из комбинаторики

Как решать уравнения из комбинаторики

Комбинации и бином ньютона

Пусть дано множество из трёх элементов: Как решать уравнения из комбинаторикиЕго двухэлементных подмножеств (не упорядоченных) существует всего три: Как решать уравнения из комбинаторикиГоворят, что существует 3 комбинации из трёх элементов по два. Пишут: Как решать уравнения из комбинаторики

Комбинацией из Как решать уравнения из комбинаторики элементов по Как решать уравнения из комбинаторики называют любое Как решать уравнения из комбинаторикиэлементное подмножество Как решать уравнения из комбинаторикиэлементного множества.

Число комбинаций из Как решать уравнения из комбинаторикиэлементов по Как решать уравнения из комбинаторикиобозначают Как решать уравнения из комбинаторикиВ отличие от размещений, комбинации — подмножества неупорядоченные.

Сравните: Как решать уравнения из комбинаторикиПри тех же значениях Как решать уравнения из комбинаторикизначение Как решать уравнения из комбинаторикименьше Как решать уравнения из комбинаторикиМожно также указать, во сколько раз меньше. Каждую Как решать уравнения из комбинаторикиэлементную комбинацию можно упорядочить Как решать уравнения из комбинаторикиспособами. В результате из одной комбинации получают Как решать уравнения из комбинаторикиразмещений (упорядоченных подмножеств) из тех же элементов. Итак,

число Как решать уравнения из комбинаторикиэлементных комбинаций в Как решать уравнения из комбинаторикираз меньше числа размещений из тех же Как решать уравнения из комбинаторикиэлементов.

То есть, Как решать уравнения из комбинаторикиотсюда

Как решать уравнения из комбинаторики

Пример №32

Вычислите: Как решать уравнения из комбинаторики

Решение:

Как решать уравнения из комбинаторики

Обратите внимание! Как решать уравнения из комбинаторикиПолагают также, что Как решать уравнения из комбинаторикидля любого Как решать уравнения из комбинаторики

Пример №33

Сколькими способами из 25 учеников можно выбрать на конференцию двух делегатов?

Решение:

Здесь Как решать уравнения из комбинаторикипорядок учеников не имеет значения.

Как решать уравнения из комбинаторики

Ответ. 300-ми способами.

Докажем, что для натуральных значений Как решать уравнения из комбинаторикиправильно тождество Как решать уравнения из комбинаторики

Доказательство. Пусть дано Как решать уравнения из комбинаторикиразличных элементов: Как решать уравнения из комбинаторикиВсего из них можно образовать Как решать уравнения из комбинаторикиразличных Как решать уравнения из комбинаторикиэлементных комбинаций. Это количество комбинаций вычислим другим способом. Из данных Как решать уравнения из комбинаторикиэлементов, кроме последнего Как решать уравнения из комбинаторикиможно образовать Как решать уравнения из комбинаторикикомбинаций. Остальные Как решать уравнения из комбинаторикиэлементные комбинации из всех данных элементов можно образовать, если к каждой комбинации из первых Как решать уравнения из комбинаторикиэлементов по Как решать уравнения из комбинаторикидописать элемент Как решать уравнения из комбинаторикиТаких комбинаций Как решать уравнения из комбинаторики

Следовательно, Как решать уравнения из комбинаторикиА это и требовалось доказать.

Такое комбинаторное тождество можно доказать также, воспользовавшись формулой числа комбинаций.

С комбинациями тесно связана формула бинома Ньютона. Вспомните формулу квадрата двучлена: Как решать уравнения из комбинаторики

Умножив Как решать уравнения из комбинаторикиполучим формулы:

Как решать уравнения из комбинаторики

Эти три формулы можно записать и так:

Как решать уравнения из комбинаторики

Оказывается, для каждого натурального значения Как решать уравнения из комбинаторикиправильна и общая формула:

Как решать уравнения из комбинаторики

Это тождество называют формулой бинома Ньютона. а её правую часть разложением бинома Ньютона. Бином — латинское название двучлена. Пользуясь этой формулой, возведём, например, двучлен Как решать уравнения из комбинаторикив пятую степень. Поскольку Как решать уравнения из комбинаторики

Как решать уравнения из комбинаторики

Доказать формулу бинома Ньютона можно методом математической индукции.

Доказательство. Предположим, что формула Как решать уравнения из комбинаторикиверна для некоторого натурального показателя степени Как решать уравнения из комбинаторикиПокажем, что тогда она верна и для следующего за ним значения Как решать уравнения из комбинаторики

Как решать уравнения из комбинаторики

Выражения в скобках преобразованы согласно формулы

Как решать уравнения из комбинаторики

Следовательно, если формула бинома Ньютона верна для Как решать уравнения из комбинаторикито она правильна и для Как решать уравнения из комбинаторикиДля Как решать уравнения из комбинаторикиона правильна, так как Как решать уравнения из комбинаторикиПоэтому на основе аксиомы математической индукции можно утверждать, что формула верна для любого натурального показателя Как решать уравнения из комбинаторики

Вычислять коэффициенты разложения бинома Ньютона можно не по формуле числа комбинаций, а пользуясь числовым треугольником Паскаля — своеобразным способом вычисления коэффициентов разложения бинома Ньютона Как решать уравнения из комбинаторики

Как решать уравнения из комбинаторики

Треугольник Паскаля можно продолжать как угодно далеко. Это следует из тождества Как решать уравнения из комбинаторикиЕго крайние числа — единицы, а каждое другое равно сумме двух ближайших к нему чисел сверху.

Например, прибавляя числа шестой строки (для Как решать уравнения из комбинаторикиполучим числа следующей строки (для Как решать уравнения из комбинаторикиСледовательно, Как решать уравнения из комбинаторикиОбщий член разложения бинома Как решать уравнения из комбинаторикиможно определить по формуле Как решать уравнения из комбинаторики

  • первый член — Как решать уравнения из комбинаторики
  • второй член — Как решать уравнения из комбинаторики
  • третий член — Как решать уравнения из комбинаторики

Пример №34

В турнире по шашкам приняли участие 5 девушек и 7 юношей. Каждый участник сыграл один раз с каждым другим. Сколько партий было: а) между девушками; б) между юношами; в) между юношами и девушками?

Решение:

а) Речь идёт о 2-элементных подмножествах (неупорядоченных) множества, состоящего из 5 элементов. Это — комбинации. Как решать уравнения из комбинаторики

б) Аналогично Как решать уравнения из комбинаторики

в) Воспользуемся правилом умножения. Поскольку каждой из 5 девушек предстоит сыграть с каждым из 7 юношей, возможных случаев Как решать уравнения из комбинаторики

Пример №35

Для дежурства в столовой приглашают 3-х учеников из 7 класса и 2-х учеников из 10 класса. Сколькими способами это можно сделать, если в 7 классе учится 24 ученика, а в 10 классе — 18.

Решение:

Речь идёт о неупорядоченных подмножествах двух разных множеств. Это — комбинации.
Как решать уравнения из комбинаторики
По правилу произведения имеем Как решать уравнения из комбинаторикиспособов выбрать учащихся для дежурства.

Пример №36

Сколько разных делителей имеет число 1001?

Решение:

Разложим заданное число на простые множители: Как решать уравнения из комбинаторикиЕсли число Как решать уравнения из комбинаторики— делитель числа 1001, то оно должно быть одним из чисел 7, 11,13 (три случая) или любым их произведением. Различных произведений может быть Как решать уравнения из комбинаторикиДелителем данного числа есть ещё единица. Следовательно, число 1001 имеет Как решать уравнения из комбинаторикиделителей.

Пример №37

Докажите, что выпуклый Как решать уравнения из комбинаторикиугольник имеет Как решать уравнения из комбинаторикидиагоналей.

Решение:

Отрезков, концами которых являются Как решать уравнения из комбинаторикивершин данного Как решать уравнения из комбинаторики-угольника, существует Как решать уравнения из комбинаторикиСреди них есть и Как решать уравнения из комбинаторикисторон данного Как решать уравнения из комбинаторики-угольника. Поэтому диагоналей он имеет Как решать уравнения из комбинаторикиКак решать уравнения из комбинаторики

Пример №38

Как решать уравнения из комбинаторики

Решение:

Как решать уравнения из комбинаторики

Как решать уравнения из комбинаторики

Все члены разложения бинома Ньютона Как решать уравнения из комбинаторикитакие же, как и члены разложения бинома Как решать уравнения из комбинаторикитолько их члены с чётными номерами отрицательные.

Пример №39

Найдите номер члена разложения Как решать уравнения из комбинаторикикоторый не содержит Как решать уравнения из комбинаторики

Решение:

Воспользуемся формулой общего члена разложения бинома. Имеем:

Как решать уравнения из комбинаторики

По условию задачи Как решать уравнения из комбинаторикито есть Как решать уравнения из комбинаторикиОтсюда Как решать уравнения из комбинаторикиСледовательно, не содержит Как решать уравнения из комбинаторикишестой член разложения бинома.

Видео:Уравнение, комбинаторика, сочетания, факториалы | Это how? #5Скачать

Уравнение, комбинаторика, сочетания, факториалы | Это how? #5

Элементы комбинаторики

Решение многих задач теории вероятностей требует знания элементов комбинаторики, основными понятиями которой являются перестановки, размещения и сочетания.

Определение: Перестановки — это комбинации из одних и тех же элементов, отличающиеся только порядком элементов.

Пример:

Даны три числа 1, 2, 3. Определить количество комбинаций из этих элементов, отличающиеся только порядком элементов.

Решение:

Комбинации из данных элементов, отличающиеся только порядком элементов: 123; 132; 213; 231; 321; 312. Всего таких комбинаций Как решать уравнения из комбинаторикиЕсли дано n элементов, то число перестановок Как решать уравнения из комбинаторикиO2. Размещения — это комбинации, составленные из n различных элементов по m элементов, которые отличаются либо составом элементов, либо их расположением.

Пример:

Даны три числа 1, 2, 3. Определить количество размещений из этих элементов по два, отличающиеся составом или порядком элементов.

Решение:

Комбинации из данных элементов по два, отличающиеся составом или порядком элементов: 12; 21; 23; 32; 13; 31. Всего таких комбинаций 6. Если дано n элементов, то число размещений по m элементов, которые отличаются либо составом элементов, либо их расположением: Как решать уравнения из комбинаторики

Определение: Сочетания — это комбинации, составленные из n различных элементов по m элементов, которые отличаются друг от друга хотя бы одним элементом.

Пример:

Даны три числа 1, 2, 3. Определить количество размещений из этих элементов по два, отличающиеся хотя бы одним элементом.

Решение:

Комбинации из данных элементов по два, отличающиеся хотя бы одним элементом: 12; 23; 13. Всего таких комбинаций 3. Если дано n элементов, то число сочетаний по m элементов, которые отличаются хотя бы одним элементом:Как решать уравнения из комбинаторики

Пример:

Пусть в урне находится n прономерованных шаров. Определить количество способов, которыми можно извлечь из урны эти шары один за другим.

Решение:

Число способов равно числу различных комбинаций из п элементов, отличающихся только порядком элементов, т.е. числу перестановок: Как решать уравнения из комбинаторики

Пример:

Из колоды, содержащей 36 карт, наугад вынимают 3 карты. Найти вероятность того, что среди выбранных карт окажется один туз.

Решение:

Событие А состоит в том, что среди выбранных карт окажется один туз. Это сложное событие состоит из двух событий: выбирается один туз из четырех, а две другие карты выбираются из оставшихся 32 карт. Следовательно, число случаев, благоприятствующих появлению события A, равно Как решать уравнения из комбинаторикиВсего возможных равновероятных исходов, образующих полную группу определяется числом сочетаний из 36 карт по 3 карты, т.е. Как решать уравнения из комбинаторикиТаким образом, вероятность события А равна Как решать уравнения из комбинаторики

Арифметика случайных событий

Будем считать, что все события, которые могут произойти в рамках данного эксперимента, располагаются внутри квадрата G, тогда невозможные события располагаются вне квадрата G (Рис. 2): Как решать уравнения из комбинаторики

Рис. 2. Квадрат возможных событий.

Таким образом, достоверное событие определяется внутренней частью квадрата, а невозможное — областью вне квадрата.

Определение: Суммой двух случайных событий А и В называется третье случайное событие С, которое состоит в том, что произойдет (или не произойдет) или событие А, или событие В : С = А + В (Рис. 3).

Определение: Суммой n случайных событий Как решать уравнения из комбинаторикиназывается случайное событие С, которое реализуется в данном опыте, если произойдет (или не произойдет) или одно событий Как решать уравнения из комбинаторики, или любая их совокупность: Как решать уравнения из комбинаторики

Как решать уравнения из комбинаторики

Рис. 3. Сумма случайных событий

Замечание: Если в словесном описании сложного события присутствует разделительный союз “или” между элементарными событиями, то речь идет о сумме этих элементарных событий.

Замечание: Суммой события А и ему противоположного события Как решать уравнения из комбинаторикиявляется достоверное событие Как решать уравнения из комбинаторикит.е. Как решать уравнения из комбинаторикиСледовательно, противоположное событие можно записать в виде Как решать уравнения из комбинаторики

Определение: Произведением двух случайных событий А и В называется третье случайное событие С, которое состоит в том, что произойдет (или не произойдет) и событие А, и событие В : Как решать уравнения из комбинаторики(Рис. 4). Как решать уравнения из комбинаторики

Рис. 4. Произведение случайных событий.

Определение: Произведением n случайных событий Как решать уравнения из комбинаторикиназывается случайное событие С, которое реализуется в данном опыте, если произойдет (или не произойдет) совместная реализация событий Как решать уравнения из комбинаторики

Замечание: Если в словесном описании сложного события присутствует соединительный союз “и” между элементарными событиями, то речь идет о произведении этих элементарных событий.

Пример №40

Пусть имеются передатчик и приемник. Приемник удален от передатчика недостаточно большое расстояние, при котором он может при определенных условиях не принять один из сигналов, переданных передатчиком. Пусть передатчик послал три сигнала. Определить следующие сложные события:

  • а) приемник принят только второй сигнал (событие А );
  • б) приемник принял только один сигнал (событие В);
  • в) приемник принял не менее двух сигналов (2 или 3 сигнала — событие С);
  • г) приемник не принял ни одного сигнала (событие D);
  • д) приемник принял хотя бы один сигнал (событие E).

Решение:

Обозначим через Как решать уравнения из комбинаторикиэлементарное событие, состоящее в том, что приемник принял сигнал i.

Сложное событие А состоит в том, что приемник не принял первый сигнал и принял второй сигнал, и не принял третий сигнал. Так как между элементарными событиями стоит соединительный союз “и”, то речь идет о их произведении, т.е. Как решать уравнения из комбинаторики

Сложное событие В состоит в том, что приемник принял или первый сигнал, или принял второй сигнал, или принял третий сигнал. Так как между элементарными событиями стоит разделительный союз “или”, то речь идет о сумме сложных событии, т.е. Как решать уравнения из комбинаторики

Рассуждая аналогично, получим выражения для остальных событий: Как решать уравнения из комбинаторикиСложное событие Е содержит в своем словесном описании слова “хотя бы один”, следовательно, оно противоположно событию, содержащему в своем словесном описании слова “ни один”, т.е. событию D: Как решать уравнения из комбинаторики

Теорема сложения вероятностей несовместных событий

Теорема: Если случайные события А и В несовместны, то вероятность их суммы равна сумме вероятностей этих событий, т.е. Р(А + В) = Р(А) + Р(В)

Доказательство: Пусть в данном опыте имеется n равновозможных, элементарных, несовместных событий и пусть в m случаях наступает событие А, а в l случаях-событие В. Тогда появлению события А + В благоприятствует m+l исходов. Поэтому Как решать уравнения из комбинаторики

Следствие: Если имеется N событий, то Как решать уравнения из комбинаторики

Следствие: Если события Как решать уравнения из комбинаторики(Как решать уравнения из комбинаторики) образуют полную группу, то Как решать уравнения из комбинаторики

Доказательство: Так как события Как решать уравнения из комбинаторикиобразуют полную группу равно возможных, элементарных, несовместных событий, то их сумма есть достоверное событие Как решать уравнения из комбинаторикиа вероятность достоверного события равна 1.

Следствие: Вероятность суммы противоположных событий равна 1.

Доказательство: В силу того, что события А и ему противоположное событие Как решать уравнения из комбинаторикиобразуют полную группу несовместных событий, то по следствию вероятность их суммы равна 1.

Замечание: Если сложное событие состоит из суммы элементарных событий, то перед применением теоремы надо определить совместны или несовместны элементарные события.

Пример:

Пусть в урне находится 5 белых шаров, 3 — красных и 4 — зеленых. Из урны наудачу вынули шар. Какова вероятность того, что данный шар цветной?

Решение:

Событие, состоящее в том, что из урны извлечен красный шар, обозначим через А. Событие, состоящее в том, что из урны извлечен зеленый шар, обозначим через В. Тогда извлечение цветного шара есть событие С. Так как события А и В несовместны, т.е. событие С состоит в том, что из урны извлечен или событие А , или событие В, то С = А + В. Используя теорему о сложении вероятностей несовместных событий, получим:

Как решать уравнения из комбинаторики

Зависимые и независимые события. Условная и безусловная вероятности

Определение: Случайные события А и В называются независимыми, если появление одного из них не влияет на вероятность появления другого события, в противном случае события называются зависимыми.

Замечание: В этом определении речь идет не о причинно-следственной связи между событиями, а о вероятностной (появление одного из них не влияет на вероятность появления другого события), которая является более общей зависимостью между событиями.

Пример №41

В хранилище находится 10 исправных и 5 неисправных приборов, причем неизвестно, какие из них исправные, а какие — нет. Обозначим событием А — из хранилища взят исправный прибор, а В — взят неисправный прибор. Пусть вначале взят неисправный прибор. Определить вероятности указанных событий с возвращением неисправного прибора на склад и без возвращения неисправного прибора в хранилище.

Решение:

Если неисправный прибор возвращается в хранилище, то события А и В независимы и их вероятности равны Как решать уравнения из комбинаторикиВо втором случае, когда неисправный прибор не возвращается на склад, общее количество приборов в хранилище изменилось и стало равным 14, причем неисправных приборов будет храниться 4. Следовательно, произошедшее событие В изменило вероятности события А и В: Как решать уравнения из комбинаторикит.е. при такой организации эксперимента события А и В являются зависимыми.

Определение: Вероятность случайного события называется безусловной, если при ее вычислении на комплекс условий, в которых рассматривается это случайное событие, не накладывается никаких дополнительных ограничений. Безусловная вероятность обозначается Как решать уравнения из комбинаторики

Определение: Вероятность случайного события называется условной, если она вычисляется при условии, что произошло другое случайное событие. Условная вероятность обозначается Как решать уравнения из комбинаторики

Теорема умножения вероятностей

Т.2. Вероятность совместного появления двух случайных событий А и В равна произведению вероятности одного из них на условную вероятность другого события, вычисленную при условии, что первое событие имело место: Как решать уравнения из комбинаторики

Доказательство: Пусть событие А состоит в том, что брошенная точка наугад в квадрат G попадает в область А, которая имеет площадь Как решать уравнения из комбинаторикиСобытие В состоит в том, что брошенная наугад в квадрат G точка попадает в область В с площадью Как решать уравнения из комбинаторикиПусть весь квадрат имеет площадь S, а область совместного наступления событий Как решать уравнения из комбинаторикиимеет площадь Как решать уравнения из комбинаторики(Рис. 5). Тогда вероятность события А равна Как решать уравнения из комбинаторикиа события В — Как решать уравнения из комбинаторики

Как решать уравнения из комбинаторики

Рис. 5. Совместное наступление зависимых и независимых случайных событий.

Вероятность совместного наступления событий Как решать уравнения из комбинаторики.Условные вероятности того, что произойдут указанные события, определяются по формулам: Как решать уравнения из комбинаторикиТаким образом, можно записать, что вероятность совместного наступления событий Как решать уравнения из комбинаторикиравна:

Как решать уравнения из комбинаторики

Замечание: Если события А и В независимы, то Как решать уравнения из комбинаторикит.е. безусловная и условная вероятности равны между собой.

В связи с вышеприведенным замечанием теорема об умножении вероятностей независимых случайных событий имеет вид:

ТЗ. Вероятность совместного наступления независимых событий равна произведению вероятностей этих событий: Как решать уравнения из комбинаторики

Замечание: Независимость случайных событий всегда взаимная. Если Как решать уравнения из комбинаторикито по теореме Как решать уравнения из комбинаторикиоткуда следует, чтоКак решать уравнения из комбинаторики

Следствие: Методом математической индукции теоремы легко обобщается на произведение N зависимых событий:

Как решать уравнения из комбинаторикиа теорема — для независимых событий: Как решать уравнения из комбинаторики

Замечание: Если сложное событие представляется в виде произведения элементарных событий, то при вычислении вероятности такого события надо определить, зависимы или независимы эти элементарные события.

Видео:Комбинаторика. Основные формулы (перестановки, сочетания, размещения) и примеры решения задач.Скачать

Комбинаторика. Основные формулы (перестановки, сочетания, размещения) и примеры решения задач.

Что такое комбинаторика

Понятие множества и его элементов:

  • Элемент а принадлежит множеству АКак решать уравнения из комбинаторикиКак решать уравнения из комбинаторики
  • Элемент Как решать уравнения из комбинаторикипринадлежит множеству Как решать уравнения из комбинаторикиКак решать уравнения из комбинаторики
  • В множестве нет элементовКак решать уравнения из комбинаторикиКак решать уравнения из комбинаторики

Множество можно представить как совокупность некоторых объектов, объединенных по определенному признаку. В математике множество — одно из основных неопределяемых понятий. Каждый объект, принадлежащий множеству А, называется элементом этого множества. Множество, не содержащее ни одного элемента, называется пустым множеством и обозначается Как решать уравнения из комбинаторики.

ПодмножествоКак решать уравнения из комбинаторики

Как решать уравнения из комбинаторики

Если каждый элемент множества А является элементом множества В, то говорят, что множество А является подмножеством множества В,

и записывают так: Как решать уравнения из комбинаторикиИспользуется также запись Как решать уравнения из комбинаторикиесли множество А или является подмножеством множества В, или равно множеству В.

Как решать уравнения из комбинаторики

Два множества называются равными, если каждый элемент первого множества является элементом второго множества и, наоборот, каждый элемент второго множества является элементом первого множества.

Пересечение множествКак решать уравнения из комбинаторики

Как решать уравнения из комбинаторики

Пересечением множеств A и В называют их общую часть, то есть множество С всех элементов, принадлежащих как множеству А, так и множеству В

Объединение множеств Как решать уравнения из комбинаторики

Как решать уравнения из комбинаторики

Объединением множеств А и В называют множество С, состоящее из всех элементов, принадлежащих хотя бы одному из этих множеств (А или В)

Разность множеств Как решать уравнения из комбинаторики

Как решать уравнения из комбинаторики

Разностью множеств А и В называется множество С, которое состоит из всех элементов, принадлежащих множеству А и не принадлежащих множеству В

Как решать уравнения из комбинаторики

Если все рассматриваемые множества являются подмножествами некоторого универсального множества U, то разность U А называется дополнением множества А. Другими словами, дополнением множества А называется множество, состоящее из всех элементов, не принадлежащих множеству А (но принадлежащих универсальному множеству).

Объяснение и обоснование:

Понятие множества

Одним из основных понятий, которые используются в математике, является понятие множества. Для него не дается определения. Можно пояснить, что множеством называют произвольную совокупность объектов, а сами объекты — элементами данного множества. Так, можно говорить о множестве учеников в классе (элементы — ученики), множестве дней недели (элементы — дни недели), множестве натуральных делителей числа 6 (элементы — числа 1, 2, 3, 6) и т. д.

В курсах алгебры и алгебры и начал анализа чаще всего рассматривают множества, элементами которых являются числа, и поэтому их называют числовыми множествами.

Как правило, множества обозначают прописными буквами латинского алфавита. Например, если множество М состоит из чисел 1; 2; 3, то его обозначают так: М = . Тот факт, что число 2 входит в это множество (является элементом данного множества М) записывается с помощью специального значка Как решать уравнения из комбинаторикиследующим образом: Как решать уравнения из комбинаторики; а то, что число 5 не входит в это множество (не является элементом данного множества), записывается так:Как решать уравнения из комбинаторики

Можно рассматривать также множество, не содержащее ни одного элемента, — пустое множество.

Например: множество простых делителей числа 1 — пустое множество.

Для некоторых множеств существуют специальные обозначения. Так, пустое множество обозначается символомКак решать уравнения из комбинаторики, множество всех натуральных чисел — буквой N, множество всех целых чисел — буквой Z, множество всех рациональных чисел — буквой Q, а множество всех действительных чисел — буквой R.

Множества бывают конечными и бесконечными в зависимости от того, какое количество элементов они содержат. Так, множества А = и М = — конечные потому, что содержат конечное число элементов, а множества N, Z, Q, R — бесконечные.

Множества задают или с помощью перечисления их элементов (это можно сделать только для конечных множеств), или с помощью описания, когда задается правило (характеристическое свойство), которое позволяет определить, принадлежит или нет данный объект рассматриваемому множеству. Например, А = (множество задано перечислением элементов), В — множество четных целых чисел (множество задано характеристическим свойством элементов множества). Последнее множество иногда записывают так: Как решать уравнения из комбинаторики— четное целое число> или так: Как решать уравнения из комбинаторики— здесь после вертикальной черточки записано характеристическое свойство.

В общем виде запись множества с помощью характеристического свойства можно обозначить так: Как решать уравнения из комбинаторики— характеристическое свойство. Например,Как решать уравнения из комбинаторики

Равенство множеств

Пусть А — множество цифр трехзначного числа 312, то есть А = , а В — множество натуральных чисел, меньших четырех, то есть В = . Поскольку эти множества состоят из одних и тех же элементов, то они считаются равными. Это записывают так: А = В.

Для бесконечных множеств таким способом (сравнивая все элементы) установить их равенство невозможно. Поэтому в общем случае равенство множеств определяется следующим образом.

Два множества называются равными, если каждый элемент первого множества является элементом второго множества и, наоборот, каждый элемент второго множества является элементом первого множества.

Из приведенного определения равенства множеств следует, что в множестве одинаковые элементы не различаются. Действительно, например, = , поскольку каждый элемент первого множества (1 или 2) является элементом второго множества и, наоборот, каждый элемент второго множества (1 или 2) является элементом первого. Поэтому, записывая множество, чаще всего каждый его элемент записывают только один раз.

Подмножество

Если каждый элемент множества А является элементом множества В, то говорят, что множество А является подмножеством множества В.

Это записывают следующим образом: Как решать уравнения из комбинаторики

Например, Как решать уравнения из комбинаторики(поскольку любое натуральное число — целое), Как решать уравнения из комбинаторики(поскольку любое целое число — рациональное), Как решать уравнения из комбинаторики(поскольку любое рациональное число — действительное).

Полагают, что всегдаКак решать уравнения из комбинаторики, то есть пустое множество является подмножеством любого множества.

Иногда вместо записи Как решать уравнения из комбинаторикииспользуется также запись Как решать уравнения из комбинаторики, если множество А является подмножеством множества В или равно множеству В. Например, можно записать, что Как решать уравнения из комбинаторики.

Сопоставим определение равенства множеств с определением подмножества. Если множества А и В равны, то: 1) каждый элемент множества А является элементом множества В, следовательно, А — подмножество ВКак решать уравнения из комбинаторики; 2) каждый элемент множества В является элементом множества А, следовательно, В — подмножество Как решать уравнения из комбинаторикиТаким образом,

два множества равны, если каждое из них является подмножеством другого.

А = В означает то же, что Как решать уравнения из комбинаторики

Иногда соотношения между множествами удобно иллюстрировать с помощью кругов (которые часто называют кругами Эйлера-Венна). Например, рисунок 118 иллюстрирует определение подмножества, а рисунок 119-отношения между множествами Как решать уравнения из комбинаторики

Как решать уравнения из комбинаторики

Операции над множествами

Над множествами можно выполнять определенные действия: находить их пересечение, объединение, разность. Дадим определение этих операций и проиллюстрируем их с помощью кругов.

Пересечением множеств А и В называют их общую часть, то есть множество С всех элементов, принадлежащих как множеству А, так и множеству В.

Пересечение множеств обозначают знаком Как решать уравнения из комбинаторики(на рисунке 120 приведена иллюстрация и символическая запись определения пересечения множеств).

Например, если А = , В = , то Как решать уравнения из комбинаторики

Объединением множеств А и В называют множество С, состоящее из всех элементов, принадлежащих хотя бы одному из этих множеств (А или В).

Объединение множеств обозначают знаком U (на рисунке 121 приведена иллюстрация и символическая запись определения объединения множеств).

Например, для множеств А и В из предыдущего примера Как решать уравнения из комбинаторикиЕсли обозначить множество иррациональных чисел через М, то М U Q = R. Разностью множеств А и В называется множество С, состоящее из всех элементов, которые принадлежат множеству А и не принадлежат множеству В.

Разность множеств обозначают знаком . На рисунке 122 приведена иллюстрация и символическая запись определения разности множеств.

Например, если А = , В = , то АВ = , а В А = . Если В — подмножество А, то разность А В называют дополнением множества В до множества А (рис. 123).

Например, если обозначить множество иррациональных чисел через М, то R Q = М: множество М иррациональных чисел дополняет множество Q рациональных чисел до множества R всех действительных чисел.

Все множества, которые мы рассматриваем, являются подмножествами некоторого так называемого универсального множества U. Его обычно изображают в виде прямоугольника, а все остальные множества — в виде кругов внутри этого прямоугольника (рис. 124). Разность U А называется дополнением множества А. Как решать уравнения из комбинаторики

Как решать уравнения из комбинаторики

Дополнением множества А называется множество, состоящее из всехэлементов, не принадлежащих множеству А (но принадлежащих универсальному множеству U).

Дополнение множества А обозначается Как решать уравнения из комбинаторики(можно читать: «А с чертой»). Например, если U = R и А = [0; 1], то Как решать уравнения из комбинаторикиДля этого примера удобно использовать традиционную иллюстрацию множества действительных чисел на числовой прямой (рис. 125).

Видео:Математика без Ху!ни. Теория вероятностей, комбинаторная вероятность.Скачать

Математика без Ху!ни. Теория вероятностей, комбинаторная вероятность.

Комбинаторика и Бином Ньютона

Элементы комбинаторики:

Комбинаторика — раздел математики, в котором изучаются способы выбора и размещения элементов некоторого конечного множества на основании некоторых условий. Выбранные (или выбранные и размещенные) группы элементов называются Соединения с повторениямими.

Если все элементы полученного множества разные — получаем соединения без повторений, а если в полученном множестве элементы повторяются, то получаем соединения с повторениями*.

Перестановкой из п элементов называется любое упорядоченное множество из Как решать уравнения из комбинаторикиэлементов.

Иными словами, это такое множество, для которого указано, какой элемент находится на первом месте, какой — на втором. какой — на п-м.

*Формулы для нахождения количества соединений с повторениями являются обязательными только для классов физико-математического профиля. Формула числа перестановок Как решать уравнения из комбинаторики Как решать уравнения из комбинаторики(читается: «Эн факториал»)

Количество различных шестизначных чисел, которые можно составить из цифр 1, 2, 3, 4, 5, 6, не повторяя эти цифры в одном числе, равно Как решать уравнения из комбинаторики

Размещением из Как решать уравнения из комбинаторикиэлементов по Как решать уравнения из комбинаторикиназывается любое упорядоченное множество из Как решать уравнения из комбинаторикиэлементов, состоящее из элементов Как решать уравнения из комбинаторики-элементного множества Формула числа размещенийКак решать уравнения из комбинаторики

Как решать уравнения из комбинаторики

Количество различных трехзначных чисел, которые можно составить из цифр 1,2,3, 4, 5, 6, если цифры не могут повторяться, равно Как решать уравнения из комбинаторики

Сочетанием без повторений из Как решать уравнения из комбинаторикиэлементов по Как решать уравнения из комбинаторикиназывается любое Как решать уравнения из комбинаторики-элементное подмножество Как решать уравнения из комбинаторики-элементного множества Формула числа сочетанийКак решать уравнения из комбинаторики

Как решать уравнения из комбинаторики(по определению считают, что Как решать уравнения из комбинаторики)

Из класса, состоящего из 25 учащихся, можно выделить 5 учащихся для дежурства по школе Как решать уравнения из комбинаторикиспособами, то есть Как решать уравнения из комбинаторикиспособами. Некоторые свойства числа сочетаний без повторений Как решать уравнения из комбинаторики

Схема решения комбинаторных задач

Если элемент А можно выбрать Как решать уравнения из комбинаторикиспособами, а элемент В — Как решать уравнения из комбинаторикиспособами, то А или В можно выбрать Как решать уравнения из комбинаторикиспособами.

Если элемент А можно выбрать Как решать уравнения из комбинаторикиспособами, а после этого элемент В — Как решать уравнения из комбинаторикиспособами, то А и В можно выбрать Как решать уравнения из комбинаторикиспособами. Выбор формулы

Учитывается ли порядок следования элементов в соединении?

Все ли элементы входят в соединение?

без повторений с повторениями без повторений с повторениями без повторений с повторениямиКак решать уравнения из комбинаторики

Объяснение и обоснование:

Понятие соединения

При решении многих практических задач приходится выбирать из определенной совокупности объектов элементы, имеющие те или иные свойства, размещать эти элементы в определенном порядке и т. д. Поскольку в этих задачах речь идет о тех или иных комбинациях объектов, то такие задачи называют комбинаторными. Раздел математики, в котором рассматриваются методы решения комбинаторных задач, называется комбинаторикой. В комбинаторике рассматривается выбор и размещение элементов некоторого конечного множества на основании определенных условий.

Выбранные (или выбранные и размещенные) группы элементов называют соединениями. Если все элементы полученного множества разные — получаем размещения без повторений, а если в полученном множестве элементы могут повторяться, то получаем размещения с повторениями. Рассматриваются соединения без повторений, а соединения с повторениями.

Решение многих комбинаторных задач базируется на двух основных правилах — правиле суммы и правиле произведения.

Правило суммы

Если на тарелке лежит 5 груш и 4 яблока, то выбрать один фрукт (то есть грушу или яблоко) можно 9 способами (5 + 4 = 9). В общем виде имеет место такое утверждение:

  • если элемент А можно выбрать Как решать уравнения из комбинаторикиспособами, а элемент В — Как решать уравнения из комбинаторикиспособами, то А или В можно выбрать Как решать уравнения из комбинаторикиспособами.

Правило произведения

Если в киоске продают ручки 5 видов и тетради 4 видов, то выбрать набор из ручки и тетради (то есть пару — ручка и тетрадь) можно 5 • 4 = 20 способами (поскольку с каждой из 5 ручек можно взять любую из 4 тетрадей). В общем виде имеет место такое утверждение:

  • если элемент А можно выбрать m способами, а после этого элемент В — Как решать уравнения из комбинаторикиспособами, то А и В можно выбрать m • п способами.

Это утверждение означает, что если для каждого из т элементов А можно взять в пару любой из Как решать уравнения из комбинаторикиэлементов В, то количество пар равно произведению Как решать уравнения из комбинаторики

Повторяя приведенные рассуждения несколько раз (или, иначе говоря, используя метод математической индукции), получаем, что правила суммы и произведения можно применять при выборе произвольного конечного количества элементов.

Следовательно, если приходится выбирать или первый элемент, или второй, или третий и т. д. элемент, количества способов выбора каждого еле-мента складывают, а когда приходится выбирать набор, в который входят и первый, и второй, и третий, и т. д. элементы, количества способов выбора каждого элемента перемножают.

Упорядоченные множества

При решении комбинаторных задач приходится рассматривать не только множества, в которых элементы можно записывать в любом порядке, но и так называемые упорядоченные множества. Для упорядоченных множеств существенным является порядок следования их элементов, то есть то, какой элемент записан на первом месте, какой на втором и т. д. В частности, если одни и те же элементы записать в разном порядке, то мы получим различные упорядоченные множества. Чтобы различить записи упорядоченного и неупорядоченного множеств, элементы упорядоченного множества часто записывают в круглых скобках, например Как решать уравнения из комбинаторики

Рассматривая упорядоченные множества, следует учитывать, что упорядоченность не является свойством самого неупорядоченного множества (из которого мы получили упорядоченное), поскольку одно и то же множество можно по-разному упорядочить. Например, множество из трех чисел можно упорядочить по возрастанию: (-5; 1; 3), по убыванию: (3; 1; — 5), по возрастанию абсолютной величины числа: (1; 3; -5) и т. д.

Будем понимать, что для того чтобы задать конечное упорядоченное множество из п элементов, достаточно указать, какой элемент находится на первом месте, какой на втором, . какой на п-м.

Размещения

Размещением из Как решать уравнения из комбинаторикиэлементов по Как решать уравнения из комбинаторикиназывается любое упорядоченное множество из Как решать уравнения из комбинаторикиэлементов, состоящее из элементов Как решать уравнения из комбинаторики-элементного множества.

Например, из множества, содержащего три цифры , можно составить следующие размещения из двух элементов без повторений: (1;5),(1;7),(5; 7), (5; 1), (7; 1), (7; 5).

Количество размещений из Как решать уравнения из комбинаторикиэлементов по Как решать уравнения из комбинаторикиобозначается Как решать уравнения из комбинаторики(читается: «А из Как решать уравнения из комбинаторикипо Как решать уравнения из комбинаторики», А — первая буква французского слова arrangement, что означает «размещение, приведение в порядок»). Как видим,Как решать уравнения из комбинаторики

Выясним, сколько всего можно составить размещений из Как решать уравнения из комбинаторикиэлементов по Как решать уравнения из комбинаторикибез повторений. Составление размещения представим себе как последовательное заполнение Как решать уравнения из комбинаторикимест, которые мы будем изображать в виде клеточек (рис. 126). На первое место мы можем выбрать один из п элементов заданного множества (то есть элемент для первой клеточки можно выбрать Как решать уравнения из комбинаторикиспособами).

Если элементы нельзя повторять, то на второе место можно выбрать только один элемент из оставшихся, то есть из Как решать уравнения из комбинаторики— 1 элементов. Теперь уже два элемента использованы и на третье место можно выбрать только один из Как решать уравнения из комбинаторики— 2 элементов и т. д. На Как решать уравнения из комбинаторики-e место можно выбрать только один из Как решать уравнения из комбинаторикиэлементов.

Поскольку требуется выбрать элементы и на первое место, и на второе, . и наКак решать уравнения из комбинаторики-e, то используем правило произведения, получим следующую формулу числа размещений из Как решать уравнения из комбинаторикиэлементов по Как решать уравнения из комбинаторикиКак решать уравнения из комбинаторики

Например, Как решать уравнения из комбинаторики(что совпадает с соответствующим значением, полученным выше). Аналогично можно обосновать формулу для нахождения числа размещений с повторениями.

При решении простейших комбинаторных задач важно правильно выбрать формулу, по которой будут проводиться вычисления. Для этого достаточно выяснить следующее: Как решать уравнения из комбинаторики

  • — Учитывается ли порядок следования элементов в соединении?
  • — Все ли заданные элементы входят в полученное соединение?

Если, например, порядок следования элементов учитывается и из Как решать уравнения из комбинаторикизаданных элементов в соединении используется только Как решать уравнения из комбинаторикиэлементов, то по определению — это размещение из Как решать уравнения из комбинаторикиэлементов по Как решать уравнения из комбинаторики.

Заметим, что после определения вида соединения следует также выяснить, могут ли элементы в соединении повторяться, то есть выяснить, какую формулу необходимо использовать — для количества соединений без повторений или с повторениями.

Примеры решения задач:

Пример №42

На соревнования по легкой атлетике приехала команда из 12 спортсменок. Сколькими способами тренер может определить, кто из них побежит в эстафете 4 х 100 м на первом, втором, третьем и четвертом этапах?

Решение:

Как решать уравнения из комбинаторикиКоличество способов выбрать из 12 спортсменок четырех для участия в эстафете равно количеству размещений из 12 элементов по 4 (без повторений), то есть Как решать уравнения из комбинаторики

Для выбора формулы выясняем ответы на вопросы, приведенные выше. Поскольку для спортсменок важно, в каком порядке они будут бежать, то порядок при выборе элементов учитывается. В полученное соединение входят не все 12 заданных элементов. Следовательно, соответствующее соединение — размещение из 12 элементов по 4 (без повторений, поскольку каждая спортсменка может бежать только на одном этапе эстафеты).

Пример №43

Найдите количество трехзначных чисел, которые можно составить из цифр 1, 2, 3, 4, 5, 6, 7, если цифры в числе не повторяются.

Решение:

Как решать уравнения из комбинаторикиКоличество трехзначных чисел, которые можно составить из семи цифр 1, 2, 3, 4, 5, 6, 7, равно числу размещений из 7 элементов по 3, то есть

Как решать уравнения из комбинаторики

Для выбора формулы выясняем, что для чисел, которые мы будем составлять, порядок следования цифр учитывается и не все элементы выбираются (только 3 из заданных семи). Следовательно, соответствующее соединение — размещение из 7 элементов по 3 (без повторений).

Пример №44

Найдите количество трехзначных чисел, которые можно составить из цифр 1, 2, 3, 4, 5, 6, 0, если цифры в числе не повторяются.

Выбор формулы проводится таким же образом, как и в задаче 2. Следует учесть, что если число, составленное из трех цифр, начинается цифрой О, то оно не считается трехзначным. Следовательно, для ответов на вопросы задачи можно сначала из заданных 7 цифр записать все числа, состоящие из 3 цифр (см. пример 2), а затем из количества полученных чисел вычесть количество чисел, составленных из трех цифр, но начинающих цифрой 0. В последнем случае мы фактически будем из всех цифр без нуля (их 6) составлять двузначные числа. Тогда их количество равно числу размещений из 6 элементов по 2 (см. решение).

Также можно выполнить непосредственное вычисление, последовательно заполняя три места в трехзначном числе и используя правило произведения. В этом случае удобно сделать рассуждения наглядными, изображая соответствующие разряды в трехзначном числе в виде клеточек, например, так:

  • 6 возможностей
  • 6 возможностей
  • 5 возможностей

Решение:

Как решать уравнения из комбинаторикиКоличество трехзначных чисел, которые можно составить из семи цифр (среди которых нет цифры 0), если цифры в числе не повторяются, равно числу размещений из 7 элементов по 3, то есть Как решать уравнения из комбинаторики

Но среди данных цифр есть цифра 0, с которой не может начинаться трехзначное число. Поэтому из размещений из 7 элементов по 3 необходимо исключить те размещения, в которых первым элементом является цифра 0. Их количество равно числу размещений из 6 элементов по 2, то есть Как решать уравнения из комбинаторикиСледовательно, искомое количество трехзначных чисел равноКак решать уравнения из комбинаторики

Пример №45

Решите уравнение Как решать уравнения из комбинаторики

Решение:

Как решать уравнения из комбинаторикиТогда получаем Как решать уравнения из комбинаторикиНа ОДЗ это уравнение равносильно уравнениям:Как решать уравнения из комбинаторики

Уравнения, в запись которых входят выражения, обозначающие количество соответствующих соединений из х элементов, считаются определенными только при натуральных значениях переменной х. В данном случае, чтобы выражение Как решать уравнения из комбинаторикиимело смысл необходимо выбирать натуральные значения Как решать уравнения из комбинаторики(в этом случае Как решать уравнения из комбинаторикитакже существует и, конечно, Как решать уравнения из комбинаторикиДля преобразования уравнения используем соответствующие формулы:Как решать уравнения из комбинаторики

Перестановки

Перестановкой из п элементов называется любое упорядоченное множество из Как решать уравнения из комбинаторикиэлементов

Напомним, что упорядоченное множество — это такое множество, для которого указано, какой элемент находится на первом месте, какой на втором. какой на Как решать уравнения из комбинаторики

Например, переставляя цифры в числе 236 (там множество цифр уже упорядоченное), можно составить такие перестановки без повторений: (2; 3; 6), (2; 6; 3), (3; 2; 6), (3; 6; 2), (6; 2; 3), (6; 3; 2) — всего 6 перестановок*.

Количество перестановок без повторений из Как решать уравнения из комбинаторикиэлементов обозначается Как решать уравнения из комбинаторики(Р — первая буква французского слова permutation — перестановка). Как видим, Как решать уравнения из комбинаторики

Как решать уравнения из комбинаторикиФактически перестановки без повторений из Как решать уравнения из комбинаторикиэлементов являются размещениями из Как решать уравнения из комбинаторикиэлементов по Как решать уравнения из комбинаторикибез повторений, поэтому Как решать уравнения из комбинаторикиПроизведение 1 • 2 • 3 •. • Как решать уравнения из комбинаторикиобозначается

Как решать уравнения из комбинаторики!. Поэтому полученная формула числа перестановок без повторений из Как решать уравнения из комбинаторикиэлементов может быть записана так:

Как решать уравнения из комбинаторики

*Отметим, что каждая такая перестановка определяет трехзначное число, составленное из цифр 2,3,6 так, что цифры в числе не повторяются.

Например, Как решать уравнения из комбинаторики(что совпадает с соответствующим значением, полученным выше).

С помощью факториалов формулу для числа размещений без повторений

Как решать уравнения из комбинаторики

можно записать в другом виде. Для этого умножим и разделим выражение в формуле (1) на произведение Как решать уравнения из комбинаторикиПолучаем Как решать уравнения из комбинаторики

Следовательно, формула числа размещений без повторений из Как решать уравнения из комбинаторикиэлементов по Как решать уравнения из комбинаторикиможет быть записана так:

Как решать уравнения из комбинаторики

Для того чтобы этой формулой можно было пользоваться при всех значениях Как решать уравнения из комбинаторикив частности, при Как решать уравнения из комбинаторикидоговорились считать, что

Как решать уравнения из комбинаторики

Например, по формуле (2) Как решать уравнения из комбинаторики

Обратим внимание, что в тех случаях, когда значение Как решать уравнения из комбинаторики! оказывается очень большим, ответы оставляют записанными с помощью факториалов.

Например,Как решать уравнения из комбинаторики

Примеры решения задач:

Напомним, что для выбора формулы при решении простейших комбинаторных задач достаточно выяснить следующее:

  • — Учитывается ли порядок следования элементов в соединении?
  • — Все ли заданные элементы входят в полученное соединение? Если, например, порядок следования элементов учитывается и все п заданных элементов используются в соединении, то по определению это перестановки из п элементов.

Пример №46

Найдите, сколькими способами можно восемь учащихся построить в колонну по одному.

Решение:

Как решать уравнения из комбинаторикиКоличество способов равно числу перестановок из 8 элементов. То есть Как решать уравнения из комбинаторики

Для выбора соответствующей формулы выясняем ответы на вопросы, приведенные выше. Поскольку порядок следования элементов учитывается и все 8 заданных элементов выбираются, то соответствующие соединения — это перестановки из 8 элементов без повторений. Их количество можно вычислить по формуле.

Пример №47

Найдите количество разных четырехзначных чисел, которые можно составить из цифр 0, 3, 7, 9 (цифры в числе не повторяются).

Решение:

Как решать уравнения из комбинаторикиИз четырех цифр 0, 3, 7, 9, не повторяя заданные цифры, можно получить Как решать уравнения из комбинаторикиперестановок. Перестановки, начинающиеся с цифры 0, не являются записью четырехзначного числа — их количество Как решать уравнения из комбинаторики. Тогда искомое количество четырехзначных чисел равно

Как решать уравнения из комбинаторики

Поскольку порядок следования элементов учитывается и для получения четырехзначного числа надо использовать все элементы, то искомые соединения — это перестановки из 4 элементов. Их количество — Как решать уравнения из комбинаторики. При этом необходимо учесть, что в четырехзначном числе на первом месте не может стоять цифра 0. Таких чисел будет столько, сколько раз мы сможем выполнить перестановки из 3 оставшихся цифр, то есть Как решать уравнения из комбинаторики.

Пример №48

Есть десять книг, из которых четыре — учебники. Сколькими способами можно поставить эти книги на полку так, чтобы все учебники стояли рядом?

Решение:

Как решать уравнения из комбинаторикиСначала будем рассматривать учебники как одну книгу. Тогда на полке надо расставить не 10, а 7 книг. Это можно сделать Как решать уравнения из комбинаторикиспособами. В каждом из полученных наборов книг можно выполнить еще Как решать уравнения из комбинаторикиперестановок учебников. По правилу умножения искомое количество способов равно Как решать уравнения из комбинаторики

Задачу можно решать в два этапа. На первом этапе условно будем считать все учебники за 1 книгу. Тогда получим 7 книг (6 не учебников + 1 условная книга — учебник). Порядок следования элементов учитывается и используются все элементы (поставить на полку необходимо все книги). Следовательно, соответствующие соединения — это перестановки из 7 элементов. Их количество — Как решать уравнения из комбинаторики.

На втором этапе решения будем переставлять между собой только учебники. Это можно сделать Как решать уравнения из комбинаторикиспособами. Поскольку нам надо переставить и учебники, и другие книги, то используем правило произведения.

Сочетания без повторений

Сочетанием без повторений из Как решать уравнения из комбинаторикиэлементов по Как решать уравнения из комбинаторикиназывается любое Как решать уравнения из комбинаторики-элементное подмножество Как решать уравнения из комбинаторики-элементного множества.

Например, из множества Как решать уравнения из комбинаторики> можно составить следующие сочетания без повторений из трех элементов: Как решать уравнения из комбинаторики

Количество сочетаний без повторений из п элементов по к элементов обозначается символом Как решать уравнения из комбинаторики(читается: «Число сочетаний из Как решать уравнения из комбинаторики» или «це из Как решать уравнения из комбинаторики», С — первая буква французского слова combinaison — сочетание). Как видим,Как решать уравнения из комбинаторики

Как решать уравнения из комбинаторикиВыясним, сколько всего можно составить сочетаний без повторений из Как решать уравнения из комбинаторикиэлементов по Как решать уравнения из комбинаторики. Для этого используем известные нам формулы числа размещений и перестановок.

Составление размещения без повторений из Как решать уравнения из комбинаторикиэлементов по Как решать уравнения из комбинаторикипроведем в два этапа. Сначала выберем Как решать уравнения из комбинаторикиразных элементов из заданного Как решать уравнения из комбинаторики-элементного множества, не учитывая порядок выбора этих элементов (то есть выберем Как решать уравнения из комбинаторики-элементное подмножество из Как решать уравнения из комбинаторики-элементного множества — сочетание без повторений из Как решать уравнения из комбинаторики-элементов по Как решать уравнения из комбинаторики). По нашему обозначению это можно сделать Как решать уравнения из комбинаторикиспособами. После этого полученное множество из к разных элементов упорядочим. Его можно упорядочить Как решать уравнения из комбинаторикиспособами. Получим размещения без повторений из Как решать уравнения из комбинаторикиэлементов по Как решать уравнения из комбинаторики. Следовательно, количество размещений без повторений из Как решать уравнения из комбинаторикиэлементов по Как решать уравнения из комбинаторикив Как решать уравнения из комбинаторикираз больше числа сочетаний без повторений из Как решать уравнения из комбинаторикиэлементов по Как решать уравнения из комбинаторики. То есть Как решать уравнения из комбинаторикиОтсюда Как решать уравнения из комбинаторикиУчитывая, что по формуле (2) Как решать уравнения из комбинаторики, получаем Как решать уравнения из комбинаторики

Например, Как решать уравнения из комбинаторикисовпадает со значением, полученным выше.

Используя формулу (3), можно легко обосновать свойство 1 числа сочетаний без повторений, приведенное в таблице 21.

Как решать уравнения из комбинаторики1) Поскольку Как решать уравнения из комбинаторики

Для того чтобы формулу (4) можно было использовать и при Как решать уравнения из комбинаторики, договорились считать, чтоКак решать уравнения из комбинаторики. Тогда по формуле (4) Как решать уравнения из комбинаторики.

Если в формуле (3) сократить числитель и знаменатель наКак решать уравнения из комбинаторики, то получим формулу, по которой удобно вычислять Как решать уравнения из комбинаторикипри малых значениях Как решать уравнения из комбинаторики:

Как решать уравнения из комбинаторики

Например, Как решать уравнения из комбинаторики

Вычисление числа сочетаний без повторений с помощью треугольника Паскаля

Для вычисления числа сочетаний без повторений можно применять формулу (3): Как решать уравнения из комбинаторики, а можно последовательно вычислять соответствующие значения, пользуясь таким свойством:

Как решать уравнения из комбинаторики

Как решать уравнения из комбинаторикиДля обоснования равенства (6) найдем сумму Как решать уравнения из комбинаторикиучитывая, что Как решать уравнения из комбинаторики

Как решать уравнения из комбинаторики, следовательно,

Это равенство позволяет последовательно вычислять значения Как решать уравнения из комбинаторикис помощью специальной таблицы, которая называется треугольником Паскаля. Если считать, что Как решать уравнения из комбинаторики, то таблица будет иметь следующий вид (табл. 23).

Каждая строка этой таблицы начинается с единицы и заканчивается единицей Как решать уравнения из комбинаторики.

Если какая-либо строка уже заполнена, например, третья, то в четвертой строке надо записать на первом месте единицу. На втором месте запишем число, равное сумме двух чисел третьей строки, стоящих над ним левее и правее (поскольку по формуле (6)Как решать уравнения из комбинаторики.

Как решать уравнения из комбинаторики

На третьем месте запишем число, равное сумме двух следующих чисел третьей строки, стоящих над ним левее и правееКак решать уравнения из комбинаторики, и т. д. (а на последнем месте снова запишем единицу).

Примеры решения задач:

Обратим внимание, что, как и раньше, для выбора формулы при решении простейших комбинаторных задач достаточно ответить на вопросы:

  1. Учитывается ли порядок следования элементов в соединении?
  2. Все ли заданные элементы входят в полученное соединение?

Для выяснения того, что заданное соединение является сочетанием, достаточно ответить только на первый вопрос. Если порядок следования элементов не учитывается, то по определению это сочетания из Как решать уравнения из комбинаторикиэлементов по Как решать уравнения из комбинаторикиэлементов.

Пример №49

Из 12 членов туристической группы надо выбрать трех дежурных. Сколькими способами можно сделать этот выбор?

Решение:

Как решать уравнения из комбинаторикиКоличество способов выбрать из 12 туристов трех дежурных равно количеству сочетаний из 12 элементов по 3 (без повторений), то есть

Как решать уравнения из комбинаторики

Для выбора соответствующей формулы выясняем ответы на вопросы, приведенные выше. Поскольку порядок следования элементов не учитывается (для дежурных неважно, в каком порядке их выберут), то соответствующее соединение является сочетанием из 12 элементов по 3 (без повторений). Для вычисления можно использовать формулы (3) или (5), в данном случае применяем формулу (3):Как решать уравнения из комбинаторики

Пример №50

Из вазы с фруктами, в которой лежит 10 разных яблок и 5 разных груш, требуется выбрать 2 яблока и 3 груши. Сколькими способами можно сделать такой выбор?

Решение:

Как решать уравнения из комбинаторикиВыбрать 2 яблока из 10 можно Как решать уравнения из комбинаторикиспособами. При каждом выборе яблок груши можно выбрать способами. Тогда по правилу произведения выбор требуемых фруктов можно выполнить Как решать уравнения из комбинаторикиспособами. Получаем

Как решать уравнения из комбинаторики

Сначала отдельно выберем 2 яблока из 10 и 3 груши из 5. Поскольку при выборе яблок или груш порядок следования элементов не учитывается, то соответствующие соединения — сочетания без повторений.

Учитывая, что требуется выбрать 2 яблока и 3 груши, используем правило произведения и перемножим полученные возможности выбора яблок(Как решать уравнения из комбинаторики) и груш (Как решать уравнения из комбинаторики).

Бином Ньютона

Как решать уравнения из комбинаторики

Поскольку Как решать уравнения из комбинаторикито формулу бинома Ньютона можно записать еще и так:

Как решать уравнения из комбинаторики

Общий член разложения степени бинома имеет вид Как решать уравнения из комбинаторики

Коэффициенты Как решать уравнения из комбинаторикиназывают биномиальными коэффициентами.

Свойства биномиальных коэффициентов:

  1. Число биномиальных коэффициентов (а следовательно, и число слагаемых в разложении Как решать уравнения из комбинаторикистепени бинома) равноКак решать уравнения из комбинаторики
  2. Коэффициенты членов, равноудаленных от начала и конца разложения, равны между собой (поскольку Как решать уравнения из комбинаторики
  3. Сумма всех биномиальных коэффициентов равна Как решать уравнения из комбинаторикиКак решать уравнения из комбинаторики
  4. Сумма биномиальных коэффициентов, стоящих на четных местах, равна сумме биномиальных коэффициентов, стоящих на нечетных местах.
  5. Для вычисления биномиальных коэффициентов можно воспользоваться треугольником Паскаля, в котором вычисления коэффициентов основываются на формуле Как решать уравнения из комбинаторики

Как решать уравнения из комбинаторики

Как решать уравнения из комбинаторики

В каждом ряду по краям стоят единицы, а каждое из остальных чисел равно сумме двух чисел, находящихся над ним справа и слева Например, Как решать уравнения из комбинаторики

Объяснение и обоснование Бинома Ньютона

Двучлен вида а + х также называют биномом. Из курса алгебры известно, что: Как решать уравнения из комбинаторики

Можно заметить, что коэффициенты разложения степени бинома Как решать уравнения из комбинаторикипри Как решать уравнения из комбинаторикисовпадают с числами в соответствующей строке треугольника Паскаля. Оказывается, что это свойство выполняется для любого натурального Как решать уравнения из комбинаторикито есть справедлива формула:

Как решать уравнения из комбинаторики

Формулу (7) называют биномом Ньютона. Правая часть этого равенства называется разложением степени бинома Как решать уравнения из комбинаторикиКак решать уравнения из комбинаторикиназывают биномиальными коэффициентами. Общий член разложения степени бинома имеет вид Как решать уравнения из комбинаторики

Как решать уравнения из комбинаторикиОбосновать формулу (7) можно, например, следующим образом.

Если раскрыть скобки в выражении Как решать уравнения из комбинаторикито есть умножить бином а + х сам на себя Как решать уравнения из комбинаторикираз, то получим многочлен Как решать уравнения из комбинаторикистепени относительно переменной х. Тогда результат можно записать так:

Как решать уравнения из комбинаторики

Чтобы найти значение Как решать уравнения из комбинаторикиподставим в обе части равенства (8) вместо х значение 0. Получаем Как решать уравнения из комбинаторикиможем записать:

Как решать уравнения из комбинаторики

Чтобы найти Как решать уравнения из комбинаторикисначала возьмем производную от обеих частей равенства (8):

Как решать уравнения из комбинаторики

затем, подставив в обе части полученного равенства (9) х = 0, получим: Как решать уравнения из комбинаторикиУчитывая, чтоКак решать уравнения из комбинаторикиможем записать: Как решать уравнения из комбинаторикиАналогично, чтобы найти Как решать уравнения из комбинаторикивозьмем производную от обеих частей равенства (9):

Как решать уравнения из комбинаторики

и, подставив х = 0 в равенство (10), получим Как решать уравнения из комбинаторикиТогда Как решать уравнения из комбинаторикиДругие коэффициенты находят аналогично. Если продифференцировать Как решать уравнения из комбинаторикираз равенство (8), то получим:

Как решать уравнения из комбинаторики

Подставляя в последнее равенство х = 0, имеем

Как решать уравнения из комбинаторики

Как решать уравнения из комбинаторики

В каждом ряду по краям стоят единицы, а каждое из остальных чисел равно сумме двух чисел, находящихся над ним справа и слева

Умножим обе части равенства (11) на Как решать уравнения из комбинаторикии найдем коэффициент

Как решать уравнения из комбинаторики. Подставляя найденные значения Как решать уравнения из комбинаторики

1, 2, . Как решать уравнения из комбинаторики) в равенство (8), получаем равенство (7).Как решать уравнения из комбинаторики

Записывая степень двучлена по формуле бинома Ньютона для небольших значений п, биномиальные коэффициенты можно вычислять по треугольнику Паскаля (табл. 25, см. также табл. 24).

Например,Как решать уравнения из комбинаторики

Так как Как решать уравнения из комбинаторикиформулу бинома Ньютона можно записать в виде:

Как решать уравнения из комбинаторики

а учитывая, чтоКак решать уравнения из комбинаторики, еще и так:

Как решать уравнения из комбинаторики

Если в формуле бинома Ньютона (12) заменить х на (-х), то получим формулу возведения в степень разности а — х:

Как решать уравнения из комбинаторики. Например, ( Как решать уравнения из комбинаторики(знаки членов разложения чередуются!).

Свойства биномиальных коэффициентов

1. Число биномиальных коэффициентов (а следовательно, и число слагаемых) в разложении Как решать уравнения из комбинаторики-й степени бинома равно Как решать уравнения из комбинаторики+ 1, поскольку разложение содержит все степени х от 0 до Как решать уравнения из комбинаторики(и других слагаемых не содержит).

2. Коэффициенты членов, равноудаленных от начала и конца разложения, равны между собой, посколькуКак решать уравнения из комбинаторики

3. Сумма всех биномиальных коэффициентов равна 2″.

Как решать уравнения из комбинаторикиДля обоснования полагаем в равенстве (13) (или в равенстве (7)) значения а = х = 1 и получаем Как решать уравнения из комбинаторики

Например, Как решать уравнения из комбинаторики

4. Сумма биномиальных коэффициентов, стоящих на четных местах, равна сумме биномиальных коэффициентов, стоящих на нечетных местах,

Как решать уравнения из комбинаторикиДля обоснования возьмем в равенстве (13) значения а =1, х = —1. Получаем

Как решать уравнения из комбинаторики

Тогда Как решать уравнения из комбинаторики

Примеры решения задач:

Пример №51

По формуле бинома Ньютона найдите разложение степени Как решать уравнения из комбинаторики

Для нахождения коэффициентов разложения можно использовать треугольник Паскаля или вычислять их по общей формуле. По треугольнику Паскаля коэффициенты равны: 1, 6, 15, 20, 15, б, 1. Учитывая, что при возведении в степень разности знаки членов разложения чередуются, получаем

Как решать уравнения из комбинаторикиДля упрощения записи ответа можно избавиться от иррациональности в знаменателях полученных выражений (см. решение) или сначала учесть, что ОДЗ заданного выражения: х > 0, и тогда Как решать уравнения из комбинаторикиТо есть заданное выражение можно записать так: Как решать уравнения из комбинаторикии возвести в степень последнее выражение.

Решение:

Как решать уравнения из комбинаторики

Пример №52

В разложении степени Как решать уравнения из комбинаторикинайти член, содержащий Как решать уравнения из комбинаторики

Решение:

► ОДЗ: Как решать уравнения из комбинаторики> 0. ТогдаКак решать уравнения из комбинаторики

Общий член разложения: Как решать уравнения из комбинаторики

По условию член разложения должен содержатьКак решать уравнения из комбинаторики, следовательно,

Как решать уравнения из комбинаторики. Отсюда Как решать уравнения из комбинаторики

Тогда член разложения, содержащий Как решать уравнения из комбинаторики, равенКак решать уравнения из комбинаторики

На ОДЗ (b > 0) каждое слагаемое в заданном двучлене можно записать как степень с дробным показателем. Это позволит проще записать общий член разложения степениКак решать уравнения из комбинаторики: Как решать уравнения из комбинаторики(где Как решать уравнения из комбинаторики= 0, 1, 2, . Как решать уравнения из комбинаторики), выяснить, какой из членов разложения содержит Как решать уравнения из комбинаторики, и записать его.

Чтобы упростить запись общего члена разложения, удобно отметить, чтоКак решать уравнения из комбинаторики

Видео:9 класс, 26 урок, Комбинаторные задачиСкачать

9 класс, 26 урок, Комбинаторные задачи

Зачем нужна комбинаторика

Для решения задач с использованием классического определения вероятности необходимо знать основные правила и формулы комбинаторики -раздела математики, изучающего методы решения комбинаторных задач — т.е. задач, связанных с подсчетом числа различных комбинаций.

Пусть Как решать уравнения из комбинаторики— элементы конечного множества. Сформулируем два важных правила, часто применяемых при решении комбинаторных задач.

Правило суммы

Если элемент Как решать уравнения из комбинаторикиможет быть выбран Как решать уравнения из комбинаторикиспособами, элемент / Как решать уравнения из комбинаторикиспособами, . элемент Как решать уравнения из комбинаторикиспособами, то выбор одного из элементов Как решать уравнения из комбинаторикиможет быть осуществлен пКак решать уравнения из комбинаторикиспособами.

Пример №53

В группе 30 студентов. Известно, что 5 из них на экзамене по математике получили оценку «отлично», 10 — оценку «хорошо», остальные -«удовлетворительно». Сколько существует способов выбрать одного студента, получившего на экзамене оценку «отлично» или «хорошо»?

Решение:

Студент, получивший оценку «отлично» может быть выбранКак решать уравнения из комбинаторикиспособами, оценку «хорошо» — Как решать уравнения из комбинаторикиспособами. По правилу суммы существует Как решать уравнения из комбинаторикиспособов выбора одного студента, получившего на экзамене оценку «отлично» или «хорошо». Как решать уравнения из комбинаторики

Правило произведения

Если элемент Как решать уравнения из комбинаторикиможет быть выбран Как решать уравнения из комбинаторикиспособами, после этого элемент Как решать уравнения из комбинаторикиможет быть выбран Как решать уравнения из комбинаторикиспособами после каждого такого выбора элемент Как решать уравнения из комбинаторикиможет быть выбран Как решать уравнения из комбинаторикиспособами, то выбор всех элементов Как решать уравнения из комбинаторикив указанном порядке может быть осуществлен Как решать уравнения из комбинаторикиспособами.

Пример №54

В группе 30 студентов. Необходимо выбрать старосту, его заместителя и профорга. Сколько существует способов это сделать?

Решение:

Старостой может быть выбран любой из 30 студентов, его заместителем – любой из оставшихся 29, а профоргом – любой из оставшихся 28 студентов, т.е. Как решать уравнения из комбинаторикиПо правилу произведения общее число способов выбора старосты, его заместителя и профорга равно Как решать уравнения из комбинаторики= = 24360 способов. ◄

Пусть дано множество из n различных элементов. Из этого множества могут быть образованы подмножества из m элементов (0 ≤ m ≤n). Например, из 5 элементов a, b, c, d, e могут быть отобраны комбинации по 2 элемента – ab, bc, cd, ba и т.д., по 3 элемента – abc, cbd, cba и т.д.

Если комбинации из n элементов по m отличаются либо составом элементов, либо порядком их расположения (либо и тем и другим), то такие комбинации называют размещениями из n элементов по m. Число размещений из n элементов по m находится по формуле Как решать уравнения из комбинаторикигде n! равно произведению n первых чисел натурального ряда, т.е. n! = 1·2·…·n.

Пример №55

Сколько можно записать двузначных чисел, используя без повторения цифры от 1 до 5?

Решение:

В данном случае двузначное число является комбинацией из пяти цифр по две цифры. Поскольку числа отличаются как составом входящих в них цифр, так и порядком их расположения, то в данном случае двузначные числа являются размещениями из пяти цифр по две. Число таких размещений

Как решать уравнения из комбинаторикиЕсли комбинации из n элементов по m отличаются только с о с т а в о м элементов (порядок их расположения не имеет значения), то такие комбинации называют сочетаниями из n элементов по m.

Число сочетаний из n элементов по m находится по формуле Как решать уравнения из комбинаторики

Пример №56

Необходимо выбрать в подарок две из пяти имеющихся различных книг. Сколькими способами можно это сделать?

Решение:

Из смысла задачи следует, что порядок выбора книг не имеет значения. Здесь важен только их состав. Поэтому в данном случае комбинации книг представляют собой сочетания из 5 книг по 2. Число таких комбинаций Как решать уравнения из комбинаторикиЕсли в размещениях из n элементов по m некоторые из элементов (или все) могут оказаться одинаковыми, то такие размещения называют размещениями с повторениями из n элементов по m. Число размещений с повторениями равно Как решать уравнения из комбинаторики

Пример №57

Сколько можно записать трехзначных чисел, которые не содержат цифр 0 и 5?

Решение:

В данном случае трехзначное число является комбинацией из восьми цифр (0 и 5 не учитываются) по три цифры. При этом некоторые из цифр (или все) могут повторяться. Поэтому в данном случае трехзначные числа является размещениями с повторениями из восьми цифр по три. Число таких размещений с повторениями Как решать уравнения из комбинаторикиЕсли в сочетаниях из n элементов по m некоторые из элементов (или все) могут оказаться одинаковыми, то такие сочетания называют сочетаниями с повторениями из n элементов по m. Число сочетаний с повторениями равно Как решать уравнения из комбинаторикигде Как решать уравнения из комбинаторикиопределяется по формуле (1.6).

Пример №58

В почтовом отделении продаются открытки восьми видов. Сколькими способами можно купить в нем три открытки?

Решение:

Учитывая, что порядок выбора открыток не имеет значения, а важен только их состав, причем некоторые из открыток (или все) могут оказаться одинаковыми, искомое число способов находим по формуле числа сочетаний с повторениями Как решать уравнения из комбинаторикиЕсли комбинации из n элементов отличаются только порядком расположения элементов, то такие комбинации называют перестановками из n элементов. Число перестановок из n элементов находится по формуле Как решать уравнения из комбинаторики

Пример №59

Порядок выступления 5 участников конкурса определяется жребием. Сколько различных вариантов жеребьевки при этом возможно?

Решение:

Каждый вариант жеребьевки отличается только порядком участников конкурса, т.е. является перестановкой из 5 элементов. Их число равно Как решать уравнения из комбинаторикиЕсли в перестановках из общего числа n элементов есть k различных элементов, при этом 1-й элемент повторяется Как решать уравнения из комбинаторикираз, 2-й элемент – Как решать уравнения из комбинаторикираз, k-й элемент – Как решать уравнения из комбинаторикираз, причемКак решать уравнения из комбинаторики, то такие перестановки называют перестановками с повторениями из n элементов. Число перестановок с повторениями равно Как решать уравнения из комбинаторики

Пример №60

Сколько можно составить шестизначных чисел, состоящих из цифр 3, 5, 7, в которых цифра 3 повторяется 3 раза, цифра 5 – 2 раза, цифра 7 – 1 раз?

Решение:

Каждое шестизначное число отличается от другого порядком следования цифр (причем Как решать уравнения из комбинаторикиа их сумма равна 6), т.е. является перестановкой с повторениями из 6 элементов. Их число равно

Как решать уравнения из комбинаторики

Рекомендую подробно изучить предметы:
  1. Теория вероятностей
  2. Математическая статистика
Ещё лекции с примерами решения и объяснением:
  • Классическое определение вероятности
  • Геометрические вероятности
  • Теоремы сложения и умножения вероятностей
  • Формула полной вероятности
  • Математическая обработка динамических рядов
  • Корреляция — определение и вычисление
  • Элементы теории ошибок
  • Методы математической статистики

При копировании любых материалов с сайта evkova.org обязательна активная ссылка на сайт www.evkova.org

Сайт создан коллективом преподавателей на некоммерческой основе для дополнительного образования молодежи

Сайт пишется, поддерживается и управляется коллективом преподавателей

Whatsapp и логотип whatsapp являются товарными знаками корпорации WhatsApp LLC.

Cайт носит информационный характер и ни при каких условиях не является публичной офертой, которая определяется положениями статьи 437 Гражданского кодекса РФ. Анна Евкова не оказывает никаких услуг.

💡 Видео

9 класс. Алгебра. Решение уравнений. Элементы комбинаторики.Скачать

9 класс. Алгебра. Решение уравнений. Элементы комбинаторики.

СочетанияСкачать

Сочетания

Комбинаторика. Сочетание. 10 класс.Скачать

Комбинаторика. Сочетание. 10 класс.

Элементы комбинаторики. Правило суммы. Правило произведения. 9 класс.Скачать

Элементы комбинаторики. Правило суммы. Правило произведения. 9 класс.

✓ Комбинаторика. Свойства чисел сочетаний | Ботай со мной #132 | Борис ТрушинСкачать

✓ Комбинаторика. Свойства чисел сочетаний | Ботай со мной #132 | Борис Трушин

ОСНОВЫ КОМБИНАТОРИКИ Урок 5. Общая схема решения комбинаторных задачСкачать

ОСНОВЫ КОМБИНАТОРИКИ Урок 5. Общая схема решения комбинаторных задач

02 Комбинаторика ЗадачиСкачать

02  Комбинаторика  Задачи

✓ Бином Ньютона. Игра в слова. Числа сочетаний | Комбинаторика | Ботай со мной #057 | Борис ТрушинСкачать

✓ Бином Ньютона. Игра в слова. Числа сочетаний | Комбинаторика | Ботай со мной #057 | Борис Трушин

Сокращение дробей с факториаламиСкачать

Сокращение дробей с факториалами

Теория вероятностей #8: формула Бернулли и примеры ее использования при решении задачСкачать

Теория вероятностей #8: формула Бернулли и примеры ее использования при решении задач

Бином Ньютона. 10 класс.Скачать

Бином Ньютона. 10 класс.

Основы комбинаторикиСкачать

Основы комбинаторики

Математика без Ху!ни. Теория вероятностей. Схема БернуллиСкачать

Математика без Ху!ни. Теория вероятностей. Схема Бернулли
Поделиться или сохранить к себе: